ROSH REVIEW MUSCULOSKELETAL

Pataasin ang iyong marka sa homework at exams ngayon gamit ang Quizwiz!

A 67-year-old man with a history of gout presents with atraumatic left knee pain. Physical examination reveals an effusion with overlying warmth and erythema. There is pain with passive range of motion. He reports a history of gout in this joint in the past. What is the appropriate next step? A Arthrocentesis of the knee B Indomethacin for acute gout C Measurement of serum uric acid D Radiograph of the knee

Correct Answer ( A ) Explanation: Septic arthritis is a bacterial or fungal infection of a joint typically spread hematogenously unless there is direct bacterial contamination. The synovium is highly vascular and lacks a basement membrane making it susceptible to bacterial seeding. Certain conditions predispose individuals to septic arthritis including diabetes, sickle cell disease, immunocompromise, alcoholism or pre-existing joint disease like rheumatoid arthritis or gout. Fever is present in less than half of cases of septic arthritis so with clinical suspicion an arthrocentesis is indicated. The knee is the most common joint affected and patients have pain (especially on passive range of motion) and decreased range of motion often accompanied by warmth, erythema and fever. This patient may have an acute gouty flare, but the clinician must exclude an infection. On joint fluid analysis, the white blood cell count of a septic joint is typically > 50,000. Indomethacin (B) is a non-steroidal anti-inflammatory agent commonly used in the treatment of acute gout. Gout is arthritis caused by deposition of monosodium urate monohydrate crystals in the joint space. Acute flares involve a monoarticular arthritis with a red, hot, swollen and tender joint. Acute episodes of gout result from overproduction or decreased secretion of uric acid. However, measurement of serum uric acid (C) does not correlate with the presence or absence of an acute flare. A radiograph of the knee (D) may show chronic degenerative changes associated with gout but will not help to differentiate gouty arthritis versus septic arthritis. One Step Further Question: Where is the most common site for gout? Answer: First MTP joint (podagra).

An 8-year-old boy was playing in his father's garden-shed and stepped on a nail. At that time, his father cleansed and dressed the wound appropriately. However, seven days later, the boy presents with difficulty walking. Examination shows a lateral sole puncture wound that is associated with surrounding warmth, erythema and edema. He also has an antalgic gait. Distal neurovascular examination is normal. An AP radiograph shows a round cuboid lucency. Which of the following is the most appropriate intervention at this time? A Bone scan B Oral antibiotics C Surgical debridement D Topical antibiotic ointment

Correct Answer ( C ) Explanation: In children, acute osteomyelitis commonly occurs with hematogenous spread or direct contamination, as in stepping on a nail. Pain, edema, erythema, warmth and generalized malaise are common symptoms of acute osteomyelitis. It is also important to consider cellulitis and septic arthritis in any case of suspected osteomyelitis. Laboratory investigation includes CBC, CRP and ESR, all of which may be normal early in the disease course. Blood cultures, which reveal the causative organism in up to 50% of cases, should also be obtained. Radiographs are also recommended. Early films may only show soft tissue swelling, but are useful in ruling-out fracture or neoplasm. Follow-up radiographs around a week after the injury usually shows periosteal elevation and bone lucency, two signs of bony destruction and significant pathology. In addition to blood cultures, aspiration is recommended if one can identify involvement of a specific bone and neoplasm has been ruled-out. Intravenous antibiotics are started immediately. Surgical debridement, especially in the case of nail puncture wounds, is strongly recommended, as failure to completely eradicate infection with Pseudomonas aeruginosa, the most common organism in nail puncture wounds, can lead to chronic refractory osteomyelitis, additional joint destruction, limb-length discrepancies, limb dysfunction and pathological fracture. A bone scan (A) or MRI may be considered for unclear cases of osteomyelitis. Timely, aggressive debridement trumps bone scanning in any case of highly suspected osteomyelitis, as is evident in the above patient presentation. Intravenous, not oral or topical (B and D), antibiotics are recommended in suspected acute osteomyelitis. One Step Further Question: What is the most common infectious agent in acute osteomyelitis? Answer: Staphylococcus aureus. Rapid Review Osteomyelitis Adults: contiguous spread Children: hematogenous spread Most common: S. aureus Sickle cell: Salmonella Cat/dog bites: Pasteurella multocida Plain films: periosteal elevation or bony erosions Bone scan, MRI Long term ABX

A 35-year-old hair stylist presents to her physician for ongoing heel pain that is exacerbated by standing on her feet all day at work. She complains that the pain is worse in the morning upon getting out of bed but subsides within 30-45 minutes. On physical exam, pain is elicited by dorsifelxion of the toes. Which of the following is the most likely diagnosis? A Calcaneal bursitis B Morton neuroma C Plantar fasciitis D Tarsal tunnel syndrome

Correct Answer ( C ) Explanation: Planter fasciitis is the pain caused by degenerative irritation at the insertion of the plantar fascia on the calcaneal tuberosity. It is twice as common in women and more common in overweight individuals. It is present in a high incidence in runners; therefore it is postulated to be caused by repetitive microtrauma. Risk factors include obesity, occupations requiring prolonged standing and weight bearing, and heel spurs. Most patients report that the pain usually is most severe during the first few steps after prolonged inactivity, such as sleeping or sitting but usually resolves upon warming up within 45 minutes. Patients who are generally standing on their feet all day report that the symptoms may actually worsen by the end of the day. Physical exam maneuvers that may reproduce the pain of plantar fasciitis include passive dorsiflexion of the toes, and having the patient stand on the tiptoes and toe-walk. Radiographs typically are not necessary for diagnosing plantar fasciitis. However, to rule out a bony tumor or fracture, always consider obtaining at least a plain radiograph before administering a corticosteroid injection. Treatment is directed at decreasing the inflammation. These treatments include icing, nonsteroidal anti-inflammatory drugs, rest and activity modification, corticosteroids, botulinum toxin type A, splinting, shoe modifications, and orthoses. Reproduction of pain in the forefoot by compressing together the metatarsal heads of the second and third or the third and fourth toes suggests the presence of a Morton neuroma (C) and is not a typical finding in plantar fasciitis. Tarsal tunnel syndrome (E) can be ruled out by percussing over the tarsal tunnel behind the medial malleolus. This test produces no pain in patients with plantar fasciitis. Inflammation of the calcaneal bursae (B) is most commonly caused by repetitive overuse and cumulative trauma, as seen in runners wearing tight-fitting shoes. Inflammation of one or both of these bursae can cause pain in the posterior heel and ankle regions. One Step Further Question: How are night splints beneficial in plantar fasciitis? Answer: Night splints maintain a neutral 90° foot-leg angle and provide constant passive stretching of the Achilles tendon and plantar fascia.

A 51-year-old man presents with low back pain that has been worsening over the past several years. Radiographs of the spine demonstrate fusion of the sacroiliac joints and a "bamboo spine." Blood tests reveal that he is human leukocyte antigen B27 positive. Which of the following findings on physical exam would be most consistent with the suspected diagnosis? A Numbness in the groin area B Pain that is worse with walking and relieved with sitting CS udden onset sexual dysfunction D Tenderness over the spine and sacroiliac joints

Correct Answer ( D ) Explanation: Tenderness over the spine and sacroiliac joints is consistent with ankylosing spondylitis. This patient's HLA-B27 positive status and X-ray findings of sacroiliac fusion and "bamboo spine" suggest this disease process. "Bamboo spine" is characterized by flowing ligamentous ossification and syndesmophyte formation on radiographs. Ankylosing spondylitis typically presents with symptoms of low back or hip pain beginning before age 40 that may worsen over time. It is an inflammatory disease associated with the HLA-B27 genotype. Men are typically more often affected than women. Physical exam will show limited range of motion of the spine with flexion and extension and tenderness over the sacroiliac joints. Kyphosis may occur in individuals of advanced age and these individuals may progress to have a stooped posture. The most common distribution of pain is in the lower back and sacroiliac joints. Treatment includes nonsteroidal anti-inflammatory drugs and physical therapy to reduce pain. If patients fail conservative measures, tissue necrosis factor blockers are recommended. Numbness in the groin area (A) and sudden onset sexual dysfunction (C) are both symptoms of cauda equina syndrome. This may be accompanied by numbness or weakness in one or both of the legs, as well as bowel or bladder dysfunction. Pain that is worse with walking and relieved with sitting (B) is typically associated with spinal stenosis. One Step Further Question: What is the leading cause of death in male patients with ankylosing spondylitis? Answer: Cardiovascular disease

An elderly man presents with 5 months of bruising, back pain, progressive leg weakness and distal leg paresthesias. Examination reveals flaccid weakness and areflexia in the legs. Laboratory testing reveals hypercalcemia, thrombocytopenia, anemia and elevated serum creatinine. A lumbar radiograph shows an L4 vertebral body fracture. Which of the following is the most likely diagnosis? A Multiple myeloma B Myelomeningocele C Myelopathy D Myositis ossificans

A Multiple myeloma Correct Answer ( A ) Explanation: Multiple myeloma is a malignant proliferation of plasma cells which produces excessive monoclonal paraproteins. Symptoms include bone pain, pathologic fractures, recurrent infections, spinal cord compression, malaise, weakness, anemia, bleeding, renal failure and hypercalcemia. Patients may be asymptomatic, and diagnosis may be incidental. Presentation often includes bone pain (commonly associated with atraumatic pathologic axial spine fractures) and abnormal laboratory results (especially abnormal serum/urine protein electrophoresis, anemia and hypercalcemia). Associated findings include macular detachment, retinal hemorrhage or cotton-wool spots, ecchymoses and pallor, cardiomegaly, hepatosplenomegaly and carpal tunnel syndrome. Bone and PET scanning, as well as plain radiography and MRI, are routinely used in the diagnostic work-up. Myelomeningocele (B), a form of spina bifida, is a congenital disorder. This neural tube defect is marked by failure of closure of the posterior spinal elements. It results in open protrusion of the meninges and spinal cord. Myelopathy (C) is a general term for spinal cord injury. It has several etiologies, of which, cervical spine injury is quite common. Myelopathy is the constellation of upper motor neuron signs (spastic weakness, spasticity, hyperreflexia, positive Babinski and Hoffman's signs), paresthesias and numbness and genitourinary dysfunction. Myositis ossificans (D), also known as heterotopic ossification, involves soft tissue calcification, typically about a joint which has suffered trauma, immobilization or surgery. The quadriceps and gluteals are commonly affected sites. One Step Further Question: What is the treatment of multiple myeloma? Answer: Corticosteroids, chemotherapeutics (melphalan, bortezomib, doxorubicin), interferons, and bisphosphonates (pamidronate). Rapid Review Multiple Myeloma Patient will be elderly Complaining of back pain PE will show CRAB: hyperCalcemia, Renal insufficiency, Anemia, lytic Bone lesions/Back pain Labs will show monoclonal antibody spike X-ray will show lytic lesions Peripheral blood smear: Rouleaux formations Serum protein electrophoresis: M spike Protein electrophoresis urine analysis: Bence-Jones proteins Most commonly caused by single clone plasma cell malignancy

Which of the following patients requires radiologic imaging for her back pain? A A 26-year-old woman with unilateral costovertebral angle tenderness, fever, and dysuria B A 30-year-old woman with midline lumbar pain and tenderness, fever, and difficulty urinating C A 35-year-old woman with bilateral lumbar level back pain after lifting furniture that is worse with movement and alleviated with rest D A 40-year-old woman with lumbar-level pain and tenderness that radiates to her left anterior thigh, with a normal neurologic exam

Correct Answer ( B ) Explanation: This patient has symptoms suspicious for a spinal epidural abscess. The classic triad is severe back pain, fever, and neurological deficits. In her case, urinary retention is her deficit, which by itself is worrisome for cauda equina syndrome. The diagnostic test of choice is MRI. Epidural abscess may require evacuation by a spine surgeon. Patients are often on IV antibiotics for many weeks. However, to avoid sterilization of the abscess and facilitate isolation of the causative organism, it is recommended that antibiotic therapy be withheld in the emergency department for suspected (or even radiographically confirmed) cases. Patients at risk for spinal epidural abscess include intravenous drug users, immunocompromised patients, alcohol abusers, diabetics, patients with malignancy, and those with recent spine procedures. Pyelonephritis (A) is an infection of the renal parenchyma and upper collecting system. In an otherwise healthy, young patient, treatment can be performed on an outpatient basis with current guidelines recommending 14 days of antibiotics. Radiographic imaging is not needed unless there is suspicion of an infected kidney stone. Nonspecific back pain (C) in the absence of "red flags" (e.g., fever, neurologic deficit, IV drug use, trauma) does not require radiographic imaging and can be managed symptomatically with resumption of usual daily activities as soon as possible. Bed rest has been shown to prolong the duration of pain, as have back exercises. Most nonspecific back pain resolves in 4-6 weeks. Radiculopathy (D) or nerve impingement syndrome may be caused by a disk herniation or other irritation of the nerve root. Given the location of symptoms, compression of the L4-L5 nerve root should be suspected but without worrisome features; imaging in the emergency department is not indicated. If her symptoms do not resolve in 4-6 weeks, an MRI of the lumbosacral spine may be warranted. One Step Further Question: What age groups are at higher risk for a malignant cause of their back pain? Answer: Patients under 18 years or over 50 years of age.

Which of the following antibiotics is associated with spontaneous tendon rupture? A Amoxicillin B Doxycycline C Levofloxacin D Sulfamethoxazole

Correct Answer ( C ) Explanation: Fluoroquinolone drugs, including levofloxacin, have been associated with spontaneous tendon ruptures. Fluroquinolones are a commonly prescribed antibiotic class. The class includes ciprofloxacin, moxifloxacin and levofloxacin. The fluoroquinolone class of drugs has a number of side effects, the most serious of which are prolongation of the QTc and spontaneous tendon rupture. Tendon rupture appears to be more common in older patients. The overall risk is between 0.1 - 0.4%. These drugs are discouraged for use in pregnant women and children secondary to their effect on cartilage.

A 72-year-old woman with osteoporosis sees you for health maintenance care. She currently takes vitamin D 200 IU/day and calcium 1500 mg/day. Her most recent bone mineral density is 2.5 standard deviations below the norm. Which of the following is the most appropriate treatment regimen? A Continue calcium, continue vitamin D and add estrogen B Continue calcium, increase vitamin D and add colchicine C Increase calcium, continue vitamin D and add calcitriol D Increase vitamin D, continue calcium and add alendronate

Correct Answer ( D ) Explanation: Osteoporosis, a decrease in bone mass, can be classified as primary and secondary. Primary osteoporosis is the most common type of osteoporosis. It is more common in women than men. A person reaches peak bone density at about age 30. After that, the rate of bone loss slowly increases, while the rate of bone building decreases. Whether a person develops osteoporosis depends on the density of the bones in early life as well as health, diet, and physical activity at all ages. In women, accelerated bone loss usually begins after monthly menstrual periods stop. This happens when a woman's production of estrogen slows down (usually between the ages of 45 and 55). In men, gradual bone thinning typically starts at about 45 to 50 years of age, when a man's production of testosterone slows down. Osteoporosis usually does not have an effect on people until they are 60 or older. Women are usually affected at an earlier age than men, because they start out with lower bone mass. Secondary osteoporosis has the same symptoms as primary osteoporosis. But it occurs as a result of having certain medical conditions, such as hyperparathyroidism, hyperthyroidism, or leukemia. It may also occur as a result of taking medicines known to cause bone breakdown, such as oral or high-dose inhaled corticosteroids (if used for more than 6 months), too high a dose of thyroid replacement, or aromatase inhibitors (used to treat breast cancer). Secondary osteoporosis can occur at any age. Dual-energy X-ray absorptiometry (DEXA) scan measures loss of bone mineral density (BMD) and is the most accurate and precise method to diagnose osteoporosis. Treatment begins with prevention, and includes early adulthood adequate calcium and vitamin D intake, regular weight-bearing exercise and tobacco and alcohol avoidance. The recommended calcium intake for those aged 19-50 years is 1000 mg/day, and those aged 51 years and older, 1200mg/day. Recommendations for vitamin D intake are 400 IU/day for young adults, and 800-1200 IU/day for the elderly. Pharmaceuticals are recommended when bone mineral density scores are below 1.5 to 2 standard deviations below the young-adult-norm, based on the presence (≤1.5) or absence (≤2) of fracture risk factors. Prescription options include estrogen replacement therapy, alendronate (or other bisphosphonate), calcitonin and raloxifene (a selective estrogen receptor modulator). This patient is only taking 200 IU/day, but should be taking 800-1200 IU/day. Osteoporosis pharmaceuticals are recommended for bone mineral density scores less than 2 standard deviations below the norm. Colchicine (B) is used in the treatment of acute gouty arthritis, not osteoporosis. A woman over age 70 years should have 800-1200 mg/day of vitamin D (A), the patient in the above scenario is only taking 200 IU/day. Additionally, estrogen is no longer a first-line approach for the treatment of osteoporosis in postmenopausal women because of increased risk of breast cancer, stroke, and venous thrombosis. 1500 mg/day of calcium (C) is sufficient for a 72-year-old woman with osteoporosis. Furthermore, this patient's vitamin D supplementation needs to be increased. One Step Further Question: List some modifiable risk factors for osteoporotic fracture? Answer: Tobacco and alcohol use, body weight <127 pounds, regular use of corticosteroids or anticonvulsants, poor balance, poor eyesight, recurrent falls and low calcium intake. Rapid Review Osteoporosis Decline in bone mass with aging → ↑ bone fragility + ↑ fracture risk F > M Risk factors: alcohol, steroid use, whites, Asians Height loss Most common fracture: vertebral body compression fractures DEXA scan T-score ≤ -2.5 Prevention: weight-bearing exercises, calcium, vitamin D, smoking cessation Pharmacologic rx: bisphosphonates (1st line

A 16-year-old boy presents complaining of right ankle pain after injuring himself while playing basketball. He states that he "twisted his ankle" when he landed on another players foot trying to get a rebound. He has not been able to walk on his right foot since this occurred two hours ago and cannot do so in the ED. On examination, his pulses are intact. Strength and sensation are normal. The ankle is swollen laterally. He is tender over the lateral malleolus and at the base of the fifth metatarsal. No tenderness or swelling is noted over the proximal tibia and fibula. Which of the following is true regarding emergency department radiographic work-up of this patient? A A complete ankle series is the only indicated study B A complete foot series is the only indicated study C Both a complete ankle and a complete foot series are indicated D No radiographic imaging is indicated at this time .

Correct Answer ( C ) Explanation: The patient should undergo X-rays of both the ankle and the foot. The Ottawa Ankle Rules (OAR) are a clinical decision making tool used to help determine the need for radiographic imaging following blunt ankle injury. They apply to acute injuries that have occurred within the previous 48 hours, however are not useful in the subacute or chronic phase. The OAR were not designed to be general guidelines for foot injuries; the foot rules only apply to the midfoot and do not apply to the hindfoot or forefoot

A 44-year-old man presents with insidious and atraumatic proximal lower leg pain. It is more tender during rest than exercise. It commonly occurs at night, and doesn't seem to improve with oral analgesics. Knee testing is negative for meniscal or ligamentous injury. A screening radiograph reveals no fracture, but does show a non-speckled calcified lesion within the proximal tibia. A bone scan shows no spine or pelvic lytic lesions. Which of the following is the most likely diagnosis? A Chondrosarcoma B Enchondroma C Multiple myeloma D Osteoid osteoma

A Chondrosarcoma Correct Answer ( A ) Explanation: Primary malignant bone tumors are rare and benign bone tumors are common. However, metastatic bone tumors are very common, and skeletal metastasis must be strongly considered in any patient over 40 years of age with any bony lesion. The most common bone tumors in those over 40 years of age are metastases, chondrosarcoma, fibrosarcoma and multiple myeloma. Constant deep achy pain, especially in the nighttime hours and not relieved by rest, is the main symptom of bone malignancy. Large benign tumors may present in a similar fashion. Mass is a less common presenting complaint, but may suggest a benign condition especially if it is nontender. Chondrosarcoma and fibrosarcoma are common bone malignancies in adult metaphyseal bones. Osteosarcoma also has a metaphyseal predilection, however, it occurs with lytic and blastic lesions in the second decade, with chondrosarcoma having calcified lesions and fibrosarcoma having lytic lesions. Radiographic examination is necessary. CT is best used in evaluating benign bone lesions. MRI is better for evaluating malignant bone lesions. Bone scans are used to detect other sites of skeletal lesions. A chest radiograph and CT are usually ordered if a malignant bone tumor is suspected. Routine laboratory testing is of limited use, however, in those over 40 years of age, consider urinalysis, urine and serum protein electrophoresis and PSA testing. Benign tumors may be observed or excised, while malignant tumors are excised and treated with chemotherapy and radiotherapy. Enchondroma (B) is a benign cartilaginous tumor. It appears as speckled calcifications within the phalanges and metacarpals of young adults. Multiple myeloma (C) most commonly occurs in those over 40 years of age. Lytic lesions of the spine and pelvis are key findings. Osteoid osteoma (D) is a childhood and teenage benign bone lesion, commonly associated with night pain that is relieved with NSAIDs. One Step Further Question: What is Ewing sarcoma?

A 67-year-old woman presents with low back pain. She states that she has had pain for over 10 years but it has gotten worse recently. She also complains of pain shooting down her leg into her foot when she walks, which improves with bending forward. Which of the following disorders is affecting this patient? A Spinal stenosis B Spinal subarachnoid hemorrhage C Transverse myelitis D Vascular claudication

A Spinal stenosis Correct Answer ( A ) Explanation: This patient's presentation is most consistent with spinal stenosis; a narrowing of the spinal canal leading to radicular symptoms on exertion. Spinal stenosis is most commonly seen in older patients (age >55 years). Typically, patients present with subacute back pain and lower extremity radiculopathies. The radicular pain occurs with walking and is relieved with rest thus mimicking claudication symptoms. Bending forward at the waist causes an increase in spinal canal diameter and reduces tension on the spinal cord leading to decrease in pain. Relief of pain with forward flexion at the waist is a unique finding in spinal stenosis. Sitting may also relieve the pain. Walking uphill may be painless (patient is flexed forward) while walking downhill (back extended) increases pain. Physical examination reveals lumbar radiculopathy (frequently of multiple levels) and increased pain with extension. Claudication (D) may be confused with spinal stenosis because pain is related to exertion but claudication will not be relieved by bending forward and should worsen with walking uphill. Spinal subarachnoid hemorrhage (B) typically results from an A-V malformation or tumor. Patients will have a sudden onset of severe pain as opposed to the chronic or subacute nature of spinal stenosis. Transverse myelitis (C) is an inflammatory process of the spinal cord that presents with combined motor and sensory deficits that may be at multiple levels. One Step Further Question: What bedside test is helpful in ruling out vascular claudication? Answer: Ankle-brachial index

A factory worker sustains a crush injury to his lower left leg. He has 9/10 sharp pain, distal pallor and paresthesias. The skin is intact but swollen. Minimal passive plantar flexion reproduces significant pain. Which of the following measurements are necessary to confirm a diagnosis? A Ankle-brachial index and wedge pressure B Diastolic blood pressure and intracompartmental pressure C Mean arterial pressure and jugular venous distension D Stroke volume and heart rate

B Diastolic blood pressure and intracompartmental pressure Correct Answer ( B ) Explanation: Extremity compartments are bound by fascial planes and contain muscles, nerves and arteries. Acute compartment syndrome occurs when the vascular supply is unable to properly perfuse these structures. Cases are typically acute and traumatic, especially when long bones, such as the femur and tibia, fracture, or when there is a crush injury. Pathologically, injury causes intracompartmental pressure to build, which decreases venous outflow, which backs-up arterial inflow, which ultimately leads to ischemic necrosis in as little as 4 hours. Pain out of proportion to injury, distal paresthesias, distal paresis or paralysis and distal pallor are common presenting symptoms. A key physical finding is intolerable passive stretching of muscles about an injury site. Since necrosis is imminent if left untreated, suspected compartment syndrome needs immediate diagnosis and treatment. Compartment syndrome is confirmed when the diastolic blood pressure minus the intracompartmental pressure is ≤ 30 mm Hg or absolute compartment pressure > 30-40 mm Hg. Once confirmed, surgical fasciotomy of the compartment is essential for favorable prognosis. The wound is left open. Delayed closure or skin grafting is performed after edema has resolved. Ankle-brachial Index (A) is used to evaluate peripheral arterial disease. Wedge pressure (A) is used to evaluate intrapulmonary capillary pressure. Mean arterial pressure (C) is based on cardiac output and peripheral vascular resistance. It is considered the perfusion pressure of end organs. It is important to consider this in patients with arterial or cardiac disease, however, it is not used in diagnosing compartment syndrome. Jugular venous distension (C) is used to evaluate cardiac function, not distal extremity perfusion. Stroke volume and heart rate (D) are used to determine cardiac output when evaluating a patient with cardiac disease, not traumatic orthopedic disease. One Step Further Question: Which two anatomic compartments are the most affected in acute compartment syndrome? Answer: Anterior compartment of the leg and the volar compartment of the forearm.

A 73-year-old woman with a past medical history of osteoporosis, degenerative disc disease, and knee pain presents with a chief complaint of worsening dyspnea and recent falls. Upon physical examination you notice an excessive curve of her thoracic spine along with forward-head posture. What is the most likely diagnosis? A Spinal hyperlordosis B Spinal kyphosis C Spinal scoliosis D Spinal stenosis

B Spinal kyphosis Correct Answer ( B ) Explanation: Spinal kyphosis is the excessive convex curvature of the thoracic spine that tends to progress with age and can lead to complications such as diminished respiratory function and frequent falls, if not corrected. Patients with kyphosis have an excessive rounding of the back that creates a hunchback appearance. Their heads appear to be displaced forward, and often patients can't fully look upward. Patients may complain of back pain and stiffness. Morbidities associated with kyphosis include impaired respiratory function with restrictive respirations, poor physical function, and frequent falls that lead to increased rate of fractures. Diagnosis is made by clinical examination as well as imaging to measure the extent of spinal curvature. Treatment includes symptom management, rehabilitation to retrain the muscles, and surgical realignment. Spinal hyperlordosis (A) is the excessive inward curvature of the lumbar spine. The cervical and lumbar spines have a natural inward curvature, but when this is excessive it is known as swayback, which can lead to lower back pain. The frontal tilt of the pelvis leads to a pelvic tilt, elongating the extensor muscle groups and shortening the flexor muscle groups. Treatment includes strengthening the extensor muscle groups and symptomatic treatment for back pain. Spinal scoliosis (C) is the lateral curvature of the spine that can occur during rapid growth spurts and cause significant spinal deformity. It typically is first noticed in childhood and usually does not cause pain. Scoliosis is diagnosed by physical examination, special tests, and imaging. Treatment involves observation, bracing, and physical therapy to treat muscular imbalances and surgical interventions to realign the spine. Spinal stenosis (D) is the narrowing of the spinal canal caused by degenerative arthritis, movement of a vertebral body, or a lesion. Narrowing of the spinal canal causes impingement on the nerves, which results in pain with activity, weakness, or sensory loss. An MRI is used to make the diagnosis, and treatment includes nonsurgical options such as physical therapy and surgical options such as spinal decompression. One Step Further Question: Is spinal kyphosis more common in men or women? Answer: Women.

A 72-year-old man with metastatic prostate cancer presents with severe increasing lower back pain. His lower extremity strength is 4/5 and symmetrically weak. He has no bowel or bladder dysfunction and no perianal anesthesia. Which of the following is the most appropriate imaging test? A Bone scan B CT scan without contrast of the lumbar spine C MRI of the thoracic and lumbar spines D Radiographs of the lumbar spine

C MRI of the thoracic and lumbar spines Correct Answer ( C ) Explanation: With a history of cancer and increasing low back pain, the most worrisome diagnosis is spinal cord compression. When cancer becomes metastatic to the spine, tumor can extend into the epidural space causing compression of the spinal cord. Additionally, tumors compress the venous plexus leading to inflammation and vasogenic edema. The most common cancers associated with cord compression are breast, prostate and lung. Classic symptoms include pain, motor deficits and bowel or bladder dysfunction. Bowel and bladder symptoms are late symptoms and their absence does not rule out the possibility of cord compression. Pain is the initial symptom in 95% of patients. The gold standard for diagnosis is MRI and should include both the lumbar and thoracic spine since there are often lesions at both levels. Bone scanning (A) is helpful in identifying bony metastasis of the entire skeleton in one imaging study. The radionuclide test identifies metastases with increased blood flow or new bone formation potentially missing disease without that (e.g. multiple myeloma). The study also does not give any information about the thecal sac necessary for the diagnosis of cord compression. CT scan without contrast of the lumbar spine (B) is helpful in the evaluation of the bony architecture of the spine and identification of metastatic disease but does not give information about the cord or thecal sac. Radiographs of the lumbar spine (D) may identify metastatic disease with less sensitivity than the other modalities mentioned. Metastatic disease is necessary for cord compression, however the lower sensitivity does not allow the sole evaluation with plain films. One Step Further Question: If MRI is not available, what other radiographic modality may identify cord compression? Answer: CT Myelography. Rapid Review Prostate Cancer RFs: age (most important), African-Americans, family hx Asymptomatic until advanced Obstructive uropathy Back pain: metastases to lumbar spine (↑ ALP) PSA > 10 ng/mL Dx: needle core biopsy

A 25-year-old carpenter presents to your primary care clinic with acute, atraumatic low back pain. This is his first episode of back pain. He denies weakness, numbness, fever or bowel and bladder incontinence. Which of the following pain descriptors, gathered from a detailed history and examination, would most prompt you to order imaging on this initial visit? A Axial pain B Flexion-based pain C Night pain D Pain began after lifting a heavy object

C Night pain Correct Answer ( C ) Explanation: The evaluation of back pain begins with assessing for "red flags," such as night pain and weight loss (suspicious for tumor), fever, chills, and sweats (consider infection), acute bony tenderness (consider fracture), morning stiffness lasting > 30 minutes in young adults (consider seronegative spondyloarthropathy, and any neurologic deficit or bowel or bladder involvement (consider nerve root compromise). Acute lumbar sprain is usually the result of repeated lifting or twisting, typically occurring in young men with laborious vocation. Axial lumbar pain is typical of sprain and internal disk disruption. Radicular pain is typical of disk bulge or herniation with nerve root encroachment. Flexion-based axial lumbar pain is typical of disk pathology, whereas extension-based axial lumbar pain suggests facet joints, central stenosis or foraminal stenosis are the main pain generators. Imaging plays a limited role in acute sprain, however, it should be considered for acute lumbar pain associated with any neurologic abnormality, trauma, concerning past medical or family history, or any case involved with fever, sweats, chills, weight loss, night pain or pain at rest. Axial pain (A), flexion-based pain (B), pain began after lifting a heavy object (D) are typical of acute lumbar sprain. Acute, flexion-based, axial lumbar pain is typical of disk tear, bulge or herniation. However, a radiograph in the setting of absent "red flags", is low yielding and not recommended at the initial visit.

A 45-year-old man presents with left leg pain. He describes a sudden onset pain in his calf area while playing basketball with his son. On exam, you squeeze his right calf and elicit plantar flexion of the right foot. When you do the same with his left calf there is no plantar flexion of his left foot. You administer ibuprofen and place the left leg in a posterior splint. Which of the following is the most appropriate next step in management? A Obtain X-rays of the ankle and foot B Prescribe physical therapy C Refer to orthopedic surgery D Screen for autoimmune diseases

C Refer to orthopedic surgery This patient has an abnormal Thompson test, a common clinical test used to diagnosed Achilles tendon rupture. Achilles tendon rupture typically occurs in the setting of sudden and forced dorsiflexion during athletic activity. Patients often report an audible "pop" and sudden pain in the calf area that improves or subsides quickly, followed by decreased ability to plantar flex the foot. The diagnosis is typically clinical. On physical examination, patients may exhibit a palpable tendon defect in the lower calf area. Some plantar flexion is usually maintained, even in cases of complete tendon rupture, due to the actions of the surrounding muscles. The Thompson test is a classic maneuver to assess the Achilles tendon. The patient lies prone, with the legs flexed at the knee at 90 degrees. The examiner then squeezes the calf muscles and observes for passive plantar flexion of the foot. The injured extremity will have a weakened or no response compared to the uninjured side. Risk factors for Achilles tendon rupture include rheumatological diseases, chronic renal failure, steroid use, and recent treatment with fluoroquinolone antibiotics. Controversy remains regarding the best treatment. While early operative repair is associated with lower risk of rerupture and possibly improved functional outcome, it also has a higher rate of complications. Nonoperative management consists of a series of casts. Ultimately, the decision depends on the patient's age, activity level and preference, and is best made on a non-emergent basis in consultation with an orthopedic surgeon or sports medicine physician. Thus, initial management includes non-steroidal anti-inflammatory analgesics, crutches, non-weightbearing status, immobilization of the ankle in plantar flexion with a splint, and outpatient referral.

A 55-year-old man presents to your office with gradual onset atraumatic low back pain, leg pain, and numbness. The pain is most often bilateral, involving the buttocks and thighs and spreads distally towards the feet. The leg pain begins and worsens with walking and standing and is relieved by sitting or lying down with hips and knees drawn up in a sitting posture. Which of the following is the most likely diagnosis? A Ankylosing spondylitis B Scoliosis C Spinal stenosis D Spondylolisthesis

C Spinal stenosis Correct Answer ( C ) Explanation: Spinal stenosis is the narrowing of the spinal canal and neural foramina. The more common acquired spinal stenosis is caused by degenerative changes in the intervertebral disks, ligaments, and facet joints surrounding the lumbar canal. These degenerative changes can be caused by disk or joint disease, back surgery, and repetitive trauma. Stenosis initially becomes symptomatic at 40 to 50 years and older. Congenital narrowing of the spinal canal causes symptoms earlier in life and is uncommon. Spinal stenosis usually occurs at cervical and lumbar segments. Patients with cervical stenosis present with radiating arm pain, numbness, paresthesia, and motor weakness. The common symptoms with lumbar stenosis are gradual onset low back pain, leg pain, and numbness. The pain is most often bilateral, involving the buttocks and thighs and spreading distally toward the feet. The classic presentation is radiating leg pain (burning or cramping) that begins or worsens with walking and standing and is relieved by sitting or lying down with hips and knees drawn up in a sitting posture (neurogenic claudication). Bending forward diminishes pain. Physical examination is frequently normal but may include loss of lumbar lordosis, impairment of spinal mobility, asymmetric knee or ankle reflexes, and muscle weakness. Results of straight-leg raising are characteristically negative. Advanced imaging studies are obtained to establish and confirm the diagnosis of spinal stenosis when surgery is considered. Plain spine radiographs often reveal degenerative changes. MRI is currently the preferred modality, followed by CT scan. Ankylosing spondylitis (A) usually affects men (male/female ratio 5:1) in their 20s and 30s. It often manifests as vague, somewhat diffuse low back pain, felt generally in the buttocks or sacroiliac area but often in the lumbar area. Pain becomes more persistent and bilateral. Back stiffness after inactivity, such as on awakening, becomes more predominant and is relieved by activity or a hot shower. Idiopathic scoliosis (B) affects women more frequently then men, and is often asymptomatic in young people. The patient with scoliosis may present with apparent deformity, pain, symptoms of difficult movement, or neurologic deficits. Most people with spondylolysis or spondylolisthesis (D) are asymptomatic. For the symptomatic minority, the most common complaint is low back pain, which can range from mild to severe. Back pain is frequently described as a dull, aching pain in the back, buttocks, and posterior thigh, the characteristics of which are not distinct compared with other degenerative back disorders. One Step Further Question: What is first line therapy for spinal stenosis? Answer: Adequate pain management, physical therapy, exercise, and weight loss

A 21-year-old track-and-field athlete trips during her last race, costing her the win, and resulting in severe foot pain. She presents with erythema and edema. During examination, stabilization of the calcaneus and rotation of the forefoot results in a clicking sensation and severe dorsal foot discomfort. Toe flexion and extension is maintained in a normal, nonpainful range. You most likely suspect a dislocation of which of the following joints? A Fibulotalar B Metatarsophalangeal C Tarsometatarsal D Tibiotalar

C Tarsometatarsal --> Lisfranc; pain is located on the dorsum of the midfoot, as compared to perimalleolar ligamentous pain; a key exam finding is pain with forefoot rotation against a stabilized hindfoot (calcaneus) Explanation: Fracture-dislocation of the tarsometatarsal joints is commonly called a Lisfranc injury. These joints exist between the three cuneiforms and the cuboid proximally and the five metatarsals distally, with the key joint being the "locking" interaction between the middle cuneiform and the second metatarsal base. Common mechanisms of injury include trauma and tripping. Pain is located on the dorsum of the midfoot, as compared to perimalleolar ligamentous pain. As such, Lisfranc injuries are easily misdiagnosed as ankle sprains. A key exam finding is pain with forefoot rotation against a stabilized hindfoot (calcaneus). This maneuver is not painful in ankle sprains or ankle mortise injury, but severely painful with Lisfranc injuries. Diagnosis can be upheld when an AP radiograph reveals lateral shift of the second metatarsal off the middle cuneiform. Nondisplaced injuries are treated with non-weight bearing casting, however, any displacement necessitates surgical intervention. Metatarsophalangeal (B) injury would typically result in painful and limited toe flexion-extension. Fibulotalar (A) and tibiotalar (D) instability/malalignment injuries occur with ankle sprain, dislocation or fracture. With these injuries, forefoot rotation around the hindfoot does not commonly result in pain. One Step Further Question: What is a typical radiographic finding that supports a diagnosis of a Lisfranc dislocation-fracture? Answer: An AP view which reveals lateral shift of the second metatarsal off the middle cuneiform. Rapid Review Lisfranc Injury Definition: any fracture or dislocation of the tarsal-metatarsal joint Plantar ecchymosis Fleck sign (pathognomonic): avulsion fracture of the medial aspect of the base of the second metatarsal Weight-bearing films may be necessary Treatment: Nondisplaced: non-weight bearing casting Displaced: surgery

A 20-year-old man presents complaining of a painful swollen finger. On examination, you note the findings seen in the image above, see image . Which of the following is true regarding this condition and its treatment? A An 11-blade scalpel or 18-gauge needle should be inserted into the eponychium and paronychium parallel to the nail until pus begins to drain B Incision should be made on the palmar aspect of the finger pulp and then the wound should be packed with gauze C It is caused by herpes simplex virus and therefore incision and drainage should be avoided to prevent spread, secondary infection and delayed healing D The nail must always be removed to allow appropriate abscess drainage

Correct Answer ( A ) Explanation: A paronychia is an infection of the soft tissues surrounding of the nail root of a finger or toe. Initially a mild cellulitis develops that ultimately progresses to an abscess under the eponychium. Standard therapy includes incision and drainage, packing and possibly oral antibiotic therapy. Drainage is performed by insertion of an 11-blade scalpel or 18-gauge needle into the eponychium and parinychium parallel to the nail until pus begins to drain. The abscess cavity should be opened completely with hemostats or small scissors. The cavity is then packed and the patient is discharged with 48-hour follow-up. If not treated early, a subungual abscess can form and purulent matter accumulates under the nail itself. Only when a subungual abscess develops does the proximal nail need to be trephined or removed to allow adequate drainage of the abscess (D). This is not required for uncomplicated paronychia. A felon is an infection of the distal pulp of the finger, which often starts after trauma to the finger and can be associated with a small foreign body. If a large abscess is present incision and drainage on the palmar aspect of the finger (B) is necessary with wound packing. Herpetic whitlow (C) is an infection of the finger secondary to herpes simplex virus 1 or 2. This can be caused by autoinoculation from herpetic lesions in other locations (mouth, genitals) or can be seen in health care workers who are exposed to infected patients. Oral antiviral therapy can shorten the duration of symptoms if started early. Incision and drainage is not advised as this can lead to further spread, secondary infection and delayed healing. One Step Further Question: In children, what is the most common cause of paronychia development? Answer: Nail biting or finger sucking. Rapid Review Incision and Drainage MRSA 60-75% Simple cutaneous abscesses: ABX not indicated Small abscesses: packing not indicated Complications: bleeding, extension, neurovascular injury

A 27-year-old previously healthy man presents to your office with a complaint of low back pain. He tells you that one week ago he was moving heavy furniture when he suddenly felt his back "give out". Since that time, he has been having constant pain and decreased range of motion. Which of the following is the most appropriate therapy? A Acetaminophen B Gabapentin C Oxycodone D Prednisone

Correct Answer ( A ) Explanation: Acute low back pain is one of the most common reasons that individuals seek medical treatment in the United States and affects approximately 85% of adults at least once in their life. It is defined by duration of symptoms, which can last up to 12 weeks. Symptoms that last longer than 12 weeks indicate either subacute or chronic back pain, which is managed differently than acute low back pain. The majority of cases of acute low back pain are due to either musculoligamentous injury or degenerative changes of the spine and most patients improve within 2-4 weeks after the onset of injury, even without intervention. Red flags of low back pain include interruption in bladder or bowel function, significant trauma, fever, history of intravenous drug use or cancer, unexplained weight loss, and pain that is increased or unrelieved by rest. A complete history and physical examination including neurologic evaluation should be done on all patients presenting with back pain. Most cases of acute low back pain do not require imaging, which should be reserved for patients with red flag symptoms. First-line treatment is with acetaminophen or non-steroidal anti-inflammatory (NSAIDs) agents (e.g. naproxen or ibuprofen). Gabapentin (B) is an anticonvulsant agent used in the treatment of chronic back pain and neuropathy. It may be considered in patients with back pain lasting longer than 12 weeks. The use of opiates in the treatment of low back pain should be with caution and careful clinical judgment due to risk of side effects, misuse, and abuse. Acetaminophen and NSAIDs should be used first and the use of opiates should be limited to short-term, scheduled dosing of short-acting agents, such as prior to bedtime to help with sleep. Long-acting opiates such as oxycodone (C) should be avoided. Systemic corticosteroids, such as prednisone (D) are not recommended in the treatment of acute low back pain and may cause adverse effects including mood lability, insomnia, and poor glycemic control in patients with diabetes mellitus. One Step Further Question: True or false: Bed rest is recommended for patients with acute low back pain? Answer: False.

Which of the following wounds is a potential candidate for primary closure? A Facial laceration from a dog bite B Laceration on sole of the foot after stepping on an unknown object in a stream C Laceration over the metacarpal-phalangeal joint sustained during a fistfight D Puncture wounds on the leg as a result of a dog bite

Correct Answer ( A ) Explanation: Although dog bite wounds are high risk for infectious complications, those that are cosmetically deforming are often closed primarily, especially on the face. Dog bites are high risk for infection from organisms commonly found in the dog's mouth, which include S. aureus, P. multocida, and S. viridans. Although controversy exists over management of animal bite wounds, generally it is agreed that wounds that are cosmetically deforming (including those on the face) should be repaired primarily. All bite wounds must be thoroughly irrigated and cleansed. Wounds on the extremities and that are not as cosmetically concerning are generally left open or loosely approximated. Certain lacerations are best managed when left open or closed in a delayed fashion. These include lacerations that are heavily contaminated or have extensive associated tissue damage, human bites, animal bites that are not cosmetically significant, and contaminated lacerations on the bottom of the feet. A laceration on the plantar aspect of the foot sustained on an unknown object while walking in a stream (B) would be considered contaminated. Lacerations over the MCP joints (C) that occur during a fistfight are considered to be human bites and should not be closed primarily. Puncture wounds on the leg (D) from a dog bite are considered high risk for infection and are not cosmetically significant, therefore should not be closed primarily. One Step Further Question: What antibiotic is first-line therapy for human bite infection prophylaxis? Answer: Amoxicillin-clavulanate. Rapid Review Dog Bites Crush injuries Gangrenous wound, sepsis: C. canimorsus Amoxicillin/clavulanate

A 29-year-old man presents with progressive back pain and stiffness that started 2 months ago. The pain is worse at night and when he wakes up in the morning and improves with exercise. In the last week he has also noted pain and stiffness in the hips and ankles. Neurological examination is unremarkable. What is the likely diagnosis? A Ankylosing spondylitis B Lumbar radiculopathy C Rheumatoid arthritis D Spinal stenosis

Correct Answer ( A ) Explanation: Ankylosing spondylitis (AS) is a chronic inflammatory condition primarily affecting the spine and pelvis. There is a strong association between the presence of human leukocyte antigen (HLA) B-27 and AS. Patients often present in their 20s to 30s with a 3:1 male predilection. Symptoms consistent with AS include an insidious onset of pain and stiffness (often present for 3 months or more) that is worse at night and the morning, but improves with mild activity. Pain typically affects the lower spine and hips, but can involve peripheral joints as well, most notably the ankles and knees. Another classic finding is pain, stiffness, and tenderness at the insertion sites of ligaments and tendons to the bone (enthesitis). This is most commonly seen at the achilles tendon attachment to the calcaneus and the plantar fascia. Diagnosis is made based on history and physical findings, presence of HLA B-27, and radiographs consistent with disease (e.g. erosions or fusion of sacroiliac joints). First-line treatment is nonsteroidal anti-inflammatory medications. Symptoms of lumbar radiculopathy (B) include lower back pain with unilateral radiation to the leg. Patients often have numbness, weakness, and decreased deep tendon reflexes which reflect the particular nerve root involved. Rheumatoid arthritis (C) typically affects older women and involves primarily the peripheral joints. Spinal stenosis (D) is narrowing of the lumbar spine that results in chronic back that is worse with prolonged standing and back extension and is relieved by rest and forward flexion of the spine. One Step Further Question: What signs and symptoms are consistent with a L5 radiculopathy? Answer: Pain that radiates down the lateral aspect of the leg into the foot, weakness with foot dorsiflexion, and decreased sensation between the first and second toes.

A 45-year-old man with a history of hyperkyphosis that started in his thirties, chronic back pain and stiffness and sacroiliac pain presents to your office as a new patient for his annual exam. He has decreased chest expansion on inspiration and complains of peripheral arthritis in addition to his back pain. Today he complains of acute blurry vision in his left eye with concomitant pain and photophobia. This is his third episode of this eye complaint, though last time it was in the right eye. Which of the following is the most likely diagnosis? Ankylosing spondylitis Fibromyalgia Osteoarthritis Polymyalgia rheumatica

Correct Answer ( A ) Explanation: Ankylosing spondylitis is a disease with chronic inflammatory effects on the spine and axial skeleton. The disease process often has concomitant effects on the peripheral joints with comorbid recurrent acute uveitis , psoriasis, aortic regurgitation, restrictive airway disease and inflammatory bowel disease. Patients have low back pain and often develop a "bamboo spine" and severe hyperkyphosis. Fibromyalgia (B) is a disorder that results in chronic musculoskeletal pain at specific trigger points. It is accompanied by fatigue, psychiatric symptoms and somatic symptoms. The etiology is unclear. Osteoarthritis (C) is a common disorder of the joints that leads to degeneration of the joint structure and pain. This can affect any bony joint but does not lead to comorbid symptoms seen in ankylosing spondylitis, such as the uveitis. Polymyalgia rheumatica (D) is an inflammatory disorder characterized by symptoms focussed in the shoulder and hip girdles. It is often associated with temporal arteritis, but not with uveitis. One Step Further Question: What gene mutation coincides with ankylosing spondylitis? Answer: HLA-B27.

A 75-year-old man with a past medical history of lung cancer presents to the Emergency Department complaining of three days of low back pain, bilateral lower extremity weakness, and urinary retention. Digital rectal exam demonstrates decreased rectal tone. What is the most likely diagnosis? A Cauda equina syndrome B Herniated nucleus pulposus C Intramedullary metastases D Spinal stenosis

Correct Answer ( A ) Explanation: Cauda equina syndrome is the compression of the lumbosacral nerve roots below the level of the conus medullaris, or the most distal portion of the spinal cord. Causes include primary or metastatic tumors, spinal infections, spinal stenosis, or herniation of a lumbar disc causing narrowing of the distal spinal canal. Compression of the lumbosacral nerve roots causes neuromuscular and urogenital symptoms such as low back pain, peripheral neuropathy, bilateral lower extremity weakness, sensory paresthesia in a saddle distribution, decreased rectal tone, and bowel or bladder dysfunction (e.g., urinary retention or incontinence). Cauda equina syndrome is a medical emergency because prolonged compression of the nerve roots can lead to permanent damage. Treatment includes immediate surgical decompression. A herniated nucleus pulposus (B) is the prolapse of the intervertebral disc through a tear in the surrounding annulus fibrosus. Symptoms can range from mild to severe back pain with radicular symptoms of sensory loss and weakness throughout the lower extremities. A herniated nucleus pulposus does not typically cause bowel or bladder dysfunction or decreased rectal tone. Intramedullary metastases (C) are rare spinal cord tumors that can be associated with lung cancer. Symptoms include low back pain, unilateral lower extremity motor weakness, and possible bowel or bladder dysfunction. Spinal stenosis (D) is the narrowing of the spinal canal, which causes symptoms of back pain with radicular symptoms. While spinal stenosis can cause sensory loss and weakness, it typically does not cause saddle distribution paresthesia or bowel or bladder dysfunction. One Step Further Question: Which deep tendon reflexes are affected in cauda equina syndrome? Answer: Patellar (knee) and Achilles (ankle) reflexes. Rapid Review Cauda Equina Syndrome Patient will be someone with a history of trauma, malignancy, epidural abscess or hematoma Complaining of acute onset of lower back pain with weakness and numbness PE will show urinary retention, saddle anesthesia, decreased rectal tone Diagnosis is made by MRI or CT myelogram Most commonly caused by a herniated disc Treatment is operative decompression

A 75-year-old man with a past medical history of lung cancer, type II diabetes mellitus, and hypertension, presents to the emergency department with a chief complaint of three days of low back pain with radiation into both lower extremities, saddle distribution paresthesia, and bowel and bladder incontinence. His last A1C was 7.9. What disorder should be highest on the differential diagnosis? A Cauda equina syndrome B Diabetic neuropathy C Intramedullary metastases D Spinal stenosis

Correct Answer ( A ) Explanation: Cauda equina syndrome occurs when there is compression of the lumbosacral nerve roots below the level of the conus medullaris, or the most distal portion of the spinal cord. Causes of cauda equina syndrome include tumors or metastases, spinal infection, or herniation of a lumbar disc causing narrowing of the spinal canal. Compression of these nerve roots results in neuromuscular and urogenital symptoms such as low back pain, peripheral neuropathy, saddle sensory paresthesia, and bowel and bladder dysfunction. This is a medical emergency because if the nerve roots remain compressed, lasting damage will ensue. Treatment is targeted toward the cause of the compression and involves surgical decompression. Diabetic neuropathy (B) is a complication of uncontrolled diabetes mellitus. Initial symptoms include loss of sensation throughout the distal toes and feet with eventual extension upward in a "stocking-glove" distribution. Patients may complain of peripheral numbness and tingling as well. Bowel and bladder incontinence, however, is not typically a symptom of diabetic neuropathy and therefore in the case above this disorder should be placed lower on the differential diagnosis. Intramedullary metastases (C) is a very rare finding, but a history of lung cancer does account for roughly 50% of the reported cases. Symptoms of intramedullary metastases include low back pain, motor weakness, and possible bowel or bladder dysfunction. In comparison to cauda equina syndrome, intramedullary metastases typically presents with unilateral weakness, but cauda equina syndrome presents with bilateral symptoms. Spinal stenosis (D) is the narrowing of the spinal canal, which results in symptoms of neurogenic claudication, or pain that is exacerbated by standing or walking and relieved with sitting or lying down. While spinal stenosis causes back pain and can cause sensory loss and weakness, it typically does not cause saddle distribution paresthesia nor bowel or bladder incontinence. If patients complain of bowel or bladder incontinence in the setting of low back pain, your suspicion for cauda equina syndrome should be high. One Step Further Question: Which deep tendon reflex is affected in cauda equina syndrome? Answer: Achilles (ankle) reflex.

You diagnose a patient with myofascial pain syndrome. In an effort to decrease her overall pain levels, which of the following medications is most appropriate to prescribe? A Cyclobenzaprine B Haloperidol C Oxycontin D Pregabalin

Correct Answer ( A ) Explanation: Myofascial pain can essentially be defined as the presence of trigger points, focal distinct painful spots located in palpable taut bands of muscles. It is a common musculoskeletal cause of pain, and can be associated with local and distal sensory abnormalities as well as autonomic, sleep and mood disorders. Treatment begins with aggressive physical therapy, which includes specific treatments of myofascial release, ultrasound, spray-and-stretch techniques, posture rebalancing and specific muscle stretch-strengthen exercises. Further treatment options include cognitive-behavior therapy and trigger point injection therapy. Medications also play a role. Muscle relaxants, such as cyclobenzaprine, provide pain relief. NSAIDs, only when used in conjunction with other active treatment, are also beneficial for pain relief. Haloperidol (B) is an antipsychotic not used in the treatment of myofascial pain. Tricyclics such as amitriptyline, however, can be used in treating myofascial pain syndrome. They are used to restore normal sleep patterns, but do not directly decrease the trigger point pain. Pregabalin (D) is cleared for use in fibromyalgia, not myofascial pain syndrome. Oxycontin (C) and other opioid medications are not recommended for myofascial pain syndrome. One Step Further Question: What are the typical laboratory abnormalities found in patients with myofascial pain syndrome? Answer: There are no specific lab tests to confirm or refute a diagnosis of myofascial pain syndrome. Rapid Review Myofascial Pain Trigger points Rx: physical therapy, cognitive-behavior therapy, muscle relaxants

A 68-year-old man presents with low back pain for the last several weeks. His pain is worse after walking. After several minutes of walking, he has to sit down due to the pain. The pain eventually dissipates following rest. Extension of his lumbar spine causes increased pain that radiates down his bilateral lower extremities. What is the next best step in managing this patient? A Order magnetic resonance imaging of the lumbar spine B Perform a facet joint corticosteroid injection C Prescribe oral prednisone and analgesics D Refer to orthopedic surgery

Correct Answer ( A ) Explanation: Ordering magnetic resonance imaging (MRI) of the lumbar spine is the most appropriate step in managing a patient with suspected spinal stenosis. Spinal stenosis is typically associated with neurogenic claudication and is caused by osteoarthritis of the lumbar spine and associated impingement of the spinal canal. Neurogenic claudication is described as pain that is worse after walking for several minutes and relieved with sitting. The degenerative changes cause narrowing of the spinal canal and compression of neural structures or the spinal artery. The pain associated with spinal stenosis is typically described as being worse with back extension and relieved with sitting. Typically, spinal stenosis affects patients > 50 years of age. Treatment includes physical therapy, epidural or facet joint corticosteroid injections, and surgical treatments. Surgical options include spinal decompression, nerve decompression, and spinal fusion. Performing a facet joint corticosteroid injection (B) may be a viable treatment option, however, this patient has neurogenic claudication and should have an MRI completed before any injection is performed. Prescribing oral prednisone and analgesics (C) is not the best first step in managing this patient. A referral to orthopedic surgery (D) may be necessary if the patient has claudication symptoms or radicular symptoms that have been ongoing for > 12 weeks. One Step Further Question: What exercises can help relieve symptoms associated with spinal stenosis? Answer: Flexion-based exercises.

A 15-year-old girl with sickle cell anemia presents with a sudden onset of fever and pain in her right hand. She states she was bitten on the hand by her friend's cat the day before. Diagnostic studies confirm osteomyelitis. Which of the following antibiotics is most appropriate for this patient? A Ampicillin-sulbactam B Cefazolin C Nitrofurantoin D Penicillin G

Correct Answer ( A ) Explanation: Osteomyelitis is an infection of bone, most commonly the long bones or vertebral bodies, that results from either hematogenous spread or an extension of local infection. Symptoms of hematogenous osteomyelitis include an acute onset of fever and pain. Physical exam reveals soft tissue swelling over the affected bone. As these symptoms are similar to septic arthritis, a key physical finding to distinguish between the two is preserved range of motion in the affected joint. The diagnostic test of choice is MRI because it will show abnormalities about two weeks before plain films will. Eventually, radiographs will show periosteal elevation, erosion or sclerosis. Blood cultures are helpful in determining an etiology, but are positive in only half of acute cases. If blood cultures do not show growth, patients will need a needle biopsy and culture. Treatment for hematogenous spread osteomyelitis is 4-6 weeks of intravenous antibiotics against the causative pathogen. This patient was bitten by a cat, making Pasteurella multocida the most likely pathogen. The treatment of choice is ampicillin-sulbactam. Cefazolin (B) is a first generation cephalosporin, which has greater effectiveness against gram-positive cocci, not gram-negative coccobacillus. Nitrofurantoin (C) is often used against urinary pathogens, but may only be administered by mouth. Osteomyelitis requires intravenous administration of antibiotics, making nitrofurantoin an inappropriate choice. Penicillin G (D) is the intravenous form of penicillin, but it does not provide the needed gram-negative coverage to eliminate Pasteurella multocida. One Step Further Question: What are the likely microorganisms of a human bite? Mixed anaerobics.

An 85-year-old woman suffered a vertebral fracture after falling in the shower. Her DEXA scan revealed a T-Score of -3. Which of the following treatments is first-line therapy for her underlying bone disorder? A Alendronate B Calcitonin C Raloxifene D Teriparatide

Correct Answer ( A ) Explanation: Osteoporosis generally does not become clinically apparent until a fracture occurs. The most common fractures seen in falls in patents with osteoporosis are Colles fractures, vertebral compression fractures and hip fractures. Two thirds of vertebral fractures are painless. Dual-energy x-ray absorptiometry (DXA) is currently the criterion standard for the evaluation of bone mineral density in people with osteoporosis. All women age 65 years and older, regardless of clinical risk factors should undergo a DEXA scan to asses for bone mineral density. A T-score of -1 to -2.5 SD indicates osteopenia and a T-score of less than -2.5 SD indicates osteoporosis. Lifestyle modification for prevention of osteoporotic fractures includes increasing weight-bearing and muscle-strengthening exercise and ensuring optimum calcium and vitamin D intake as an adjunct to active anti-fracture therapy. Bisphosphonates are first line medications for the prevention and treatment of postmenopausal osteoporosis, osteoporosis in males, and glucocorticoid-induced osteoporosis. Their major pharmacologic action is the inhibition of bone resorption. Alendronate is a type of bisphosphonate that is commonly used for treatment of osteoporosis. Calcitonin (B) is used for the treatment of postmenopausal osteoporosis in women more than 5 years post menopause with low bone mass relative to healthy premenopausal females. Calcitonin-salmon injection should be reserved for patients who refuse or cannot tolerate estrogens or in whom estrogens are contraindicated. Teriparatide (D) is a parathyroid hormone, which is responsible for the primary regulation of calcium and phosphate metabolism. It can be used to treat osteoporosis however it is not a first-line agent. Raloxifene (C) is a selective estrogen receptor-modulator and it provide the beneficial effects of estrogens without the adverse effects. One Step Further Question: What are the side effects of estrogen therapy in osteoporosis? Answer: Increased risk for breast and endometrial cancer.

A 65-year-old woman presents to your office to follow up on her recent osteoporosis screening. Her dual-energy X-ray absorptiometry scan results reveal a diagnosis of osteoporosis. Which of the following is the most appropriate therapy? A Alendronate B Calcitonin C Estrogen-progestin therapy D Fluoride

Correct Answer ( A ) Explanation: Osteoporosis is a very common disease that leads to increased risk of bone fracture in those affected. Risk factors for osteoporosis include advanced age, female gender, Caucasian and Asian race, body weight less than 58 kg, cigarette smoking, increased alcohol intake, previous fracture and family history of hip fracture. All women regardless of risk factors should begin screening at age 65. Pharmacologic therapy is recommended for women with a history of vertebral or hip fracture or a dual-energy X-ray absorptiometry (DXA) scan revealing a diagnosis of osteoporosis, defined as a T-score of less than or equal to -2.5 standard deviations below the young adult female reference mean. Oral bisphosphonates including alendronate or risedronate are used as initial therapy due to their efficacy, low cost and evidence of long-term safety. Calcitonin (B) is generally not used in the treatment of osteoporosis because it is not as effective as bisphosphonates and is associated with increased cancer rates with long-term use. Estrogen-progestin therapy (C) is no longer a first line treatment for osteoporosis due to increased risk of venous thromboembolism, stroke and breast cancer. Fluoride (D) has not demonstrated a reduction in fractures and is not recommended in the treatment of osteoporosis. One Step Further Question: When should the next DXA scan be done after initiating treatment with oral bisphosphonates? Answer: In two years.

Which of the following causes secondary osteoporosis in women? ACorticosteroid use BEstrogen use CHyperlipidemia DHypoparathyroidism

Correct Answer ( A ) Explanation: Primary osteoporosis is the deterioration of bone mass that is unassociated with other chronic illness and is related to aging and decreased estrogen. Therefore, early menopause or premenopausal estrogen deficiency states may accelerate the development of primary osteoporosis. Prolonged periods of inadequate calcium intake, sedentary lifestyle and tobacco and alcohol abuse also contribute to this condition. Secondary osteoporosis results from chronic conditions that contribute significantly to accelerated bone loss. These chronic conditions include endogenous and exogenous thyroxine excess, hyperparathyroidism, malignancies, gastrointestinal diseases, medications, renal failure and connective tissue diseases. Osteoporosis is a common complication of long-term glucocorticoid therapy and is responsive to bisphosphonates. Osteoporosis is much more common in elderly women due to the decrease in estrogen supply. Fractures in the hip, lumbar spine and radius are common in osteoporosis. The most widely used technique to assess bone mineral density is dual-energy X-ray absorptiometry (DXA). Osteoporosis is the classification for a T score of equal to or greater than 2.5 standard deviations below the sex-adjusted mean for normal young adults at peak bone mass. Estrogen use (B) is actually used as a treatment in osteoporosis as it is the depletion of this hormone that cause bone density loss in elderly women. Hypoparathyroidism (C) is not a risk factor for osteoporosis, instead hyperparathyroidism is known as a secondary cause of osteoporosis. Hyperlipidemia (D) has no correlation with osteoporosis. However, obesity is a risk factor for osteoarthritis. One Step Further Question: What is a protective risk factor in osteoporosis that is a negative risk factor in osteoarthritis? Answer: Obesity

A 22-year-old woman presents with pain and swelling of the plantar surface of her foot. She reports that two days prior to arrival she was walking in sneakers and stepped on a nail that punctured her foot. Physical examination demonstrates 3 cm of warm, blanching erythema without induration or fluctuance. Her tetanus vaccination is up-to-date. An X-ray does not show any foreign body. Which antibiotic is an appropriate choice to cover the organism at risk from her shoe? A Ciprofloxacin B Dicloxacillin C Linezolid D Trimethoprim-sulfamethoxazole

Correct Answer ( A ) Explanation: Puncture wounds on the bottom of the foot are at high risk for infection and also foreign body retention. Particularly in the setting of a puncture wound that passes through the sole of a shoe, Pseudomonas aeruginosa is the causative organism requiring treatment. In theory, the organism has an affinity for rubber or plastic materials and therefore is transferred to the skin through the puncture. In general, Pseudomonas rarely causes skin and soft tissue infections. Ciprofloxacin is an oral fluoroquinolone with adequate Pseudomonas aeruginosa coverage. Care must be taken in children with ciprofloxacin because of concern over the development of tendonopathy. In considering all skin and soft tissue infections, Staph and Strep are still the most common causative organisms, but this particular clinical scenario mandates coverage for Pseudomonas. Dicloxacillin (B) is a penicillin derivative commonly used to treat infections caused by Staph aureus. Staph aureus coverage is required in skin infections given the prevalence of that organism in cellulitis infections. However, it does not cover Pseudomonas. Linezolid (C) is an oxazolidinone antibiotic with activity against Gram-positive organisms. It is typically reserved for resistant infections like Methicillin-resistant Staph aureus and Vancomycin-resistant Enterococcus. It is available as an oral agent allowing for transition to oral therapy from IV medication in the hospital. Trimethoprim-sulfamethoxazole (D) is a sulfa-based antibiotic with in the setting of skin infections has widespread activity against community-associated Methicillin-resistant Staph aureus. It does not have any activity against Pseudomonas and would not be indicated in this clinical scenario. One Step Further Question: What other classic skin infection does Pseudomonas cause? Answer: Hot tub folliculitis.

Which of the following is the most common type of shoulder dislocation? AAnterior BInferior CPosterior DSuperior

Correct Answer ( A ) Explanation: Shoulder dislocations are very common, due in part to the inherent instability of the shoulder joint. Anterior dislocations are most common and make up 95-97% of all shoulder dislocations. The mechanism of injury is generally due to a blow to the arm that is abducted, externally rotated, and extended. Sports activities such as blocking a basketball shot, motor vehicle accidents and a fall on an outstretched arm can all lead to anterior shoulder dislocation. Treatment involves reduction of the shoulder, immobilization and referral to an orthopedic specialist within one week of the injury. Inferior dislocation (B) and superior dislocation (D) are the least common types of shoulder dislocation. Posterior dislocation (C) is the second most common type of shoulder dislocation, but only occurs 2-4% of the time. Posterior dislocation can be seen with a blow to the anterior shoulder or post-seizure due to violent muscle contractions. One Step Further Question: Which nerve is most often injured with shoulder dislocations? Answer: The axillary nerve.

What medication should be first-line treatment in moderate musculoskeletal back pain? A Acetaminophen B Cyclobenzaprine C Diazepam D Oxycodone

Correct Answer ( A ) Explanation: The first choice for patients with mild to moderate back pain should be acetaminophen or a nonsteroidal anti-inflammatory drug (NSAID) like ibuprofen or naprosyn. In patients with severe pain, opiate medications may be necessary. However, it is important to note that no studies have demonstrated the superiority of one pain medication (or class of medication) over another in the treatment of back pain. Patients with milder symptoms should initially be treated with acetaminophen or an NSAID. These medications are well tolerated in short courses with minor side effects. NSAIDs have not been shown to be superior to acetaminophen. Cyclobenzaprine (B) and diazepam (C) are muscle relaxants that promote sleep but have no evidence supporting their effectiveness in the treatment of back pain. Oxycodone (D) is an opiate analgesic with significant side effects and no proven benefit over more conservative treatments. One Step Further Question: In which patients should NSAIDs be avoided? Answer: Patients with acid peptic ulcer disease, renal insufficiency, diabetes, CHF and a history of allergic reaction to NSAIDs. Rapid Review Lumbago Acute nonspecific low back pain Lack Red flags Continue daily activities as tolerated Physical therapy, NSAIDs, acetaminophen, opioids

The shoulder is most vulnerable to an anterior glenohumeral dislocation when in which of the following positions? A Abduction and external rotation B Abduction and internal rotation C Adduction and external rotation D Adduction and internal rotation

Correct Answer ( A ) Explanation: The shoulder is most vulnerable when abducted and externally rotated. A fall or tackle with the arm in this position can cause an anterior shoulder dislocation. Posterior shoulder dislocations are less common and are associated with grand mal seizures and electric shock. Abduction and internal rotation (B), adduction and external rotation (C), adduction and internal rotation (D) are not associated with increased risk of anterior shoulder dislocation. One Step Further Question: What is the most common type of major joint dislocation? Answer: Glenohumeral joint (shoulder). Rapid Review Anterior Shoulder Dislocation Subcoracoid most common Arm abducted, externally rotated Complications: axillary nerve damage, Bankart lesion, Hill-Sachs deformity

Which of the following correctly describes a positive straight leg raise? A Pain in the back which radiates past the knee when raising the symptomatic leg 45 degree B Pain in the back which radiates past the opposite affected knee when raising the asymptomatic leg 45 degrees C Pain in the back without radiation to the leg when raising the asymptomatic leg 45 degrees D Pain in the back without radiation to the leg when raising the symptomatic leg 45 degrees

Correct Answer ( A ) Explanation: The straight leg test is an easy test to assess for the presence of an L5 or S1 radiculopathy. While supine with the legs both extended, the symptomatic leg is passively raised keeping the knee straight. The presence of pain in the back which radiates past the knee when the leg is elevated 30 to 70 degrees is a positive test. The straight leg test has a sensitivity of 91%, but a specificity of only 26%. Pain in the back which radiates past the opposite affected knee when raising the asymptomatic leg 45 degrees (B) is considered a positive crossed straight leg test. This test has a much higher specificity for diagnosing sciatica (88%), but a low sensitivity. Pain in the back without radiation to the leg when raising the asymptomatic leg 45 degrees (C) does not correctly describe the straight leg raise test. Pain in the back without radiation to the leg when raising the symptomatic leg 45 degrees (D) is considered a negative finding. One Step Further Question: What is a reverse straight leg test? Answer: Pain with passive extension of the hip while the patient is prone. It is indicative of an L3 or L4 radiculopathy

A 44-year-old man presents to the emergency department with back pain. On exam, the man appears uncomfortable and is tender to palpation over the right paraspinal muscles. Which of the following characteristics would most likely necessitate further evaluation? A Back pain lasting more than ten days B History of chronic corticosteroid use C History of peptic ulcer disease D Pain radiating down the posterior thigh during straight leg test

Correct Answer ( B ) Explanation: *A history of chronic corticosteroid use is a risk factor for vertebral compression fractures* and would most likely require further evaluation. Low back pain a very common complaint seen in primary care setting and in the emergency department. Risk factors for back pain include smoking, obesity, increasing age, female gender, sedentary work, and strenuous physical work. The majority of low back pain cases are due to musculoskeletal causes. Rare, but serious systemic etiologies of back pain include cauda equina syndrome, metastatic cancer, epidural abscess, vertebral compression fracture, and vertebral osteomyelitis. In order to help rule out serious etiologies, patients who present with low back pain should be questioned about constitutional symptoms, history of malignancy, neurologic symptoms, recent bacterial infections, history of injection drugs, history of corticosteroid use, and recent history of back procedures. Physical exam should include inspection of the back, palpation of the spine, straight leg raise, and neurologic exam. Laboratory studies are not typically required for the majority of patients who present with low back pain. If underlying infection or malignancy is suspected, the erythrocyte sedimentation rate (ESR) or C-reactive protein (CRP) may be useful. Routine plain radiographs are not recommended for patients with back pain lasting less than four weeks. Plain radiographs are indicated in patients with risk factors for compression fractures. A magnetic resonance image (MRI) is indicated for patients with any neurologic deficit and in patients with a high clinical suspicion for epidural abscess. First-line pharmacologic treatment for subacute and chronic low back pain is acetaminophen or nonsteroidal anti-inflammatory drugs (NSAIDs). Second line treatment options include short-term opioids, short-term muscle relaxants, and antidepressants. Physical therapy is also beneficial in patients with subacute and chronic low back pain. Spinal fusion is the most common surgery for chronic nonspecific back pain with degenerative changes. Back pain lasting more than ten days (A) is not a "red flag" symptom. The majority of patients with low back pain improve within four weeks. Patients who do not improve within four to six weeks of conservative therapy should be reevaluated. A history of peptic ulcer disease (C) is not a "red flag" symptom. Other conditions that can radiate pain to the back include pancreatitis, pancreatic cancer, thoracic and abdominal aneurysm, pericarditis, and nephrolithiasis. Pain radiating down the posterior thigh during straight leg test (D) is indicative of radiculopathy. The straight leg raise causes increased dural tension in the low lumbar and high sacral levels. L5 radiculopathy is the most common radiculopathy of the spine. Immediate diagnostic imaging is not required for neurologically intact patients with suspected radiculopathy and who are at low risk for neoplastic or infectious etiologies. One Step Further Question: What is another name for the straight leg test? Answer: Lasègue sign

Which of the following fractures requires orthopedic referral? A First distal phalanx fracture without intra-articular involvement B First metatarsal fracture with 1 mm displacement C Midshaft fifth metatarsal midshaft with 1 mm displacement D Second metatarsal fracture with 2 mm displacement

Correct Answer ( B ) Explanation: *All first metatarsal fractures, including first metatarsal fracture with 1 mm displacement, should receive an orthopedic referral.* Metatarsal fractures are the second most common type of foot fractures in adults, behind toe fractures. Direct blows and twisting injuries are the most common mechanisms of metatarsal fracture. Greater force is typically needed to fracture the first metatarsal due to its larger size. Risk factors for metatarsal fractures include osteoporosis, diabetes, and decreased physical activity. Patients with metatarsal fractures usually present with acute onset of foot pain and difficulty walking. Physical exam is often notable swelling and ecchymosis. Point tenderness is common, but swelling may make localization difficult. The tarso-metatarsal joint should be palpated to evaluate for Lisfranc injury. Three-view plain radiographs are recommended as initial diagnostic imaging. All open fractures require emergent orthopedic referral. In non-athletes, indications for orthopedic referral include multiple metatarsal fractures, intra-articular fractures, first metatarsal fracture, or second to fifth metatarsal fracture displaced greater than 3-4 mm. Initial management for minimally displaced involves a posterior splint, non-weight-bearing status, elevation, and ice. Healing typically occurs within 6 weeks in the majority of adults with metatarsal fractures. Midshaft fifth metatarsal midshaft fracture with 1 mm displacement (C), first distal phalanx fracture without intra-articular involvement (A), and second metatarsal fracture with 2 mm displacement (D) do not require orthopedic referral. These fractures can be managed with a walking cast or air boot. Patients with nondisplaced or minimally displaced second to fifth metatarsal fractures should have radiographs repeated 5-7 days after the injury to assure the fracture remains non-displaced. All first metatarsal fractures require orthopedic referral because it is a major contributor to the foot's ability to bear weight. One Step Further Question: What is a Jones fracture? Answer: Fracture of the proximal fifth metatarsal diaphysis.

A 77-year-old woman presents with 2 days of bowel incontinence, leg pain, and difficulty ambulating. Lower extremity examination reveals bilateral sensory deficits of the lumbosacral dermatomes, bilateral hyporeflexia, negative Babinski testing, and weak rectal tone. There is no lower extremity spasticity. Cranial nerve and upper extremity examination is normal. These symptoms are most likely the result of injury to which of the following structures? A Lumbar disk B Lumbosacral nerve roots C Spinal cord gray matter D Spinal cord white matter

Correct Answer ( B ) Explanation: A normal adult spinal cord ends as the conus medullaris at the L1-L2 level. The remaining L2-S4 spinal nerve roots that occupy the subdural space from the conus to the filum terminale are called the cauda equina. The cauda equina is a collection of post-ganglionic nerves (lumbosacral nerve roots), so injury to it presents as a lower motor neuron injury pattern with flaccid tone, paresis or paralysis, and hyporeflexia (or areflexia). Upper motor neuron symptoms such as hyperreflexia and clonus, spasticity and upgoing toes (positive Babinski test) are absent. The clinical presentation of cauda equina syndrome includes back pain, perianal numbness, loss of rectal tone, bowel and bladder (urinary retention followed by incontinence) abnormalities, leg weakness, loss of bulbocavernosus reflex, and loss of deep tendon reflexes. Cauda equina syndrome most commonly results from a herniated disk, but can also occur from trauma, an infection (epidural abscess), or a hematoma (epidural hematoma). A lumbar disk abnormality (A), such as herniation, typically causes unilateral radicular pain and symptoms, but it is almost exclusively associated with normal rectal tone and perineal sensation. Spinal cord tumor, or other spinal cord neuronal cell body disease affecting the gray matter (C), would cause upper motor neuron symptoms. Hyperreflexia, positive Babinski testing and spasticity are not present in this patient. Spinal cord neuronal axon or tract disease affecting the white matter (D), as in multiple sclerosis, usually presents earlier in age, and with upper extremity and cranial nerve neurologic findings, both of which are absent in this case.

A 36-year-old woman presents to your clinic with a complaint of low back pain that began 4 days ago. She was moving some heavy furniture and felt her back "give out". Physical exam reveals muscle spasm in the lumbar paraspinal muscles and decreased range of motion with back flexion, extension, and rotation. Which of the following is the most appropriate therapy? A Amitriptyline B N aproxen C Oxycodone D Prednisone

Correct Answer ( B ) Explanation: Acute low back pain is one of the most common reasons that people seek out medical care in the United States. The majority of patients with low back pain who seek out treatment in the first 3 days after onset will recover within 2 weeks. Most cases of mechanical low back pain are due to muscular strain or sprain. Other causes include degenerative changes of the spine, herniated disks, compression fractures related to osteoporosis, and spinal stenosis. Diagnosis is by history and physical exam. Imaging should be reserved for patients with clinical findings that suggest systemic disease or trauma. Serious systemic causes of low back pain include cauda equina syndrome, spinal infection and metastatic cancer. Cauda equina syndrome is generally related to a metastatic tumor within the vertebra. Clinical manifestations include pain, weakness and both bladder and bowel dysfunction. Patients with spinal epidural abscess generally present with fever, malaise and a history of intravenous drug use or recent spinal injection. In cancer patients, one of the most common sites of metastasis is to the bone. Patients with back pain should be asked about any history of cancer. Once serious etiologies have been ruled out, first-line treatment for acute low back pain is with non-steroidal anti-inflammatory agents (NSAIDs), such as naproxen or ibuprofen. Amitriptyline (A) is a tricyclic antidepressant sometimes used in the treatment of chronic back pain. Oxycodone (C) is an opioid analgesic and may provide some benefit in patients with acute low back pain whose symptoms are not relieved with NSAIDs and muscle relaxers. Due to concerns about high abuse potential and drug diversion, these agents should be used with caution and are not considered first-line therapy. Systemic corticosteroids such as prednisone (D) are not recommended in the treatment of acute low back pain due to a number of adverse effects including mood lability, insomnia, and poor glycemic control in patients with diabetes. One Step Further Question: True or false: Bed rest should be recommended to patients with acute low back pain? Answer: False.

A 14-year-old girl sprained her ankle. She rates her pain 5/10. On examination, she has moderate tenderness and swelling with decreased range of motion secondary to pain. Although quite painful, she is able to ambulate. What is the grade of this ankle sprain? A Grade I B Grade II C Grade III D Grade IV

Correct Answer ( B ) Explanation: Ankle sprains are caused by partial or complete tearing of one or more ligaments that support the ankle joint. Injuries are most often due to an inversion mechanism, causing injury to the lateral ligaments. The anterior talofibular ligament (ATFL) is the first ligament to be injured, followed by the calcaneofibular ligament (CFL), and finally, in the most severe lateral sprains, the posterior talofibular ligament (PTFL). A syndesmotic ankle sprain (high ankle sprain) involves the ligaments that connect the tibia and fibula. Ankle sprains are graded I, II or III based on exam findings and functional loss. The patient in the above scenario has a Grade II ankle sprain (incomplete tear) characterized by moderate pain, swelling, tenderness and ecchymosis with mild to moderate joint instability and some loss of range and function. Typically these are painful with weight bearing. Grade I (A) is characterized by mild stretching and microscopic tears of the ligament. Mild swelling and tenderness is noted with no joint instability. The patient is able to bear weight and ambulate with minimal pain. Due to their benign nature, these injuries are not frequently seen in the healthcare setting. Grade III (C) is characterized by a complete tear of the ligament. Severe pain, swelling, tenderness and ecchymosis is noted with significant joint instability. There is significant loss of range and function and the patient cannot bear weight or ambulate. There is no Grade IV (D) ankle sprain. One Step Further Question: If a patient has ankle instability, what physical exam tests should be positive? Answer: Anterior drawer test and talar tilt test. Rapid Review Ankle Sprain Partial or complete tearing of ≥ 1 ligaments MC due to inversion mechanism MC sprained: anterior talofibular ligament (ATFL) Grade I: microscopic tear, grade II: incomplete tear, grade III: complete tear RICE

Which of the following is true regarding the management of a subungual hematoma? A Antibiotics should be prescribed to patients after trephination of uncomplicated subungual hematomas B Complete nail removal may be necessary if the nail bed has been significantly damaged C Trephination is contraindicated when the subungual hematoma is associated with a distal phalanx fracture D Trephination is indicated only when the hematoma involves greater than 50% of the nail bed

Correct Answer ( B ) Explanation: Complete nail removal may be necessary if the damage to the nail bed is significant. A subungual hematoma is the result of a laceration to the nail bed. If the nail is stable and firmly adherent to the nail bed and the surrounding tissues are not significantly damaged, there is no reason to remove the nail and repair the underlying nailbed laceration. The nail itself will keep the nail bed laceration well approximated. Nail trephination in this scenario often results in a good cosmetic endpoint. Nail trephination is preformed under sterile technique with adherence to universal precautions. Local anesthesia is generally unnecessary for uncomplicated hematoma trephination. More complex injuries, including those in which nail removal is necessary, will require a digital block for patient comfort. Most commonly hot cautery (with a heated paperclip or disposable cautery device) is used. The device is inserted perpendicular to the nail in the center of the hematoma. A pop will be felt as the device passes through the nail and at this point the device should be removed to avoid damage to the nail bed. Blood will come out of the hole and the nail will return to a normal color. Gentle downward pressure applied by the patient will help blood escape. Occasionally large hematomas require more than one trephination site for adequate drainage. After the procedure, the area should be cleaned and dressed with a dry dressing. Splints are applied as indicated. The patient is instructed to keep the digit dry for 2 days. Alternative trephination techniques include the use of an 18-gauge needle twisted into the nail. Also a small 29-gauge needle can be inserted parallel and under the nail, past the hematoma site. This is useful for hard to drain second-fifth toe subungual hematomas. Subungual hematoma is often the result of slamming a finger in a door, striking a finger with an object or dropping an object onto a toe. This can result in a distal phalanx fracture as well. This is not a contraindication to nail trephination (C). The general rule is that all painful, acute (24-48 hours old) subungual hematomas should be drained irrespective of the size (D). Some sources cite that only hematomas greater than 50% should be drained, however smaller hematomas should be drained if the patient is experiencing significant discomfort. There is little risk of infection after trephination of an uncomplicated subungual hematoma therefore antibiotics are not prescribed (A). One Step Further Question: What is the differential diagnosis for non-traumatic subungual hematoma? Answer: Melanoma and Kaposi sarcoma

A soccer player presents to the emergency department after being kicked in the shin by his opponent. Upon presentation he demonstrates pain out of proportion as well as extreme pain with passive range of motion. You have a high suspicion for acute compartment syndrome. What laboratory finding is most closely associated with this disorder? A Aldolase B Creatine kinase C Creatine kinase MB D Erythrocyte sedimentation rate

Correct Answer ( B ) Explanation: Creatine kinase is a marker of muscle injury and if elevated in rhabdomyolysis and commonly associated with compartment syndrome. Skeletal muscle has the highest concentration of creatine kinase than any other tissue. Therefore, it is the most sensitive indicator of an injury as well as marker to follow during the course of a muscular injury. In the case of acute compartment syndrome, the muscular compartment of the lower leg is under extreme pressure, which leads to muscle injury and, eventually, tissue ischemia. Aldolase (A) is an enzyme found in skeletal muscle, the liver, and the brain. Acute compartment syndrome would cause aldolase to be elevated because there was an injury to the muscles. However, because it is found in multiple places throughout the body, it is not the most specific lab for this disorder. Creatine kinase-MB (C) is an isoenzyme form of the creatine kinase enzyme. The majority of this enzyme is found in cardiac tissue and is found to be elevated in cases of myocardial infarction or any stress to the heart muscle. Erythrocyte sedimentation rate (D) helps to assess kidney function. It may elevate in cases of compartment syndrome due to rhabdomyolysis but is a poor indicator of injury. One Step Further Question: What is another acute phase reactant that is elevated when there is an inflammatory or infectious process present? Answer: C-reactive protein

72-year-old woman presents with left knee pain for the past several months that has been worse over the last seven days. It is worse with walking and better with rest. She takes acetaminophen to alleviate the pain. She denies any injury or previous surgery on the knee. An X-ray is ordered and shows chondrocalcinosis, narrowed joint spaces, and subarticular reactive sclerosis. Bone spur formation is also noted. What is the most likely diagnosis? A Calcium pyrophosphate deposition disease B Degenerative arthritis C Neuropathic osteoarthropathy D Rheumatoid arthritis

Correct Answer ( B ) Explanation: Degenerative arthritis (or osteoarthritis) is characterized on plain film imaging by three distinct features: narrowed joint spaces, spur formation, and subarticular reactive sclerosis. The primary form of this condition is caused by "wear and tear" on the bones from the aging process. The secondary form results from injury or damage to the intra-articular cartilage, causing the bones to wear more easily. The knees in particular may show chondrocalcinosis, but mineralization of the bone is generally intact. History will generally reveal mechanical pain and physical exam may be relatively benign. It is the most common joint diseases affecting older adults and can lead to disability. Pharmacologic treatments include oral or topical NSAIDs, acetaminophen, intra-articular corticosteroid or hyaluronate injections, or opioids. Surgical intervention is the definitive treatment. Calcium pyrophosphate deposition disease (CPDD) or "pseudogout" (A) is shown radiographically as chondrocalcinosis of both hyaline and fibrocartilage and large osteophyte formation. The knee is commonly involved with these episodes, but the history of this patient does not coincide with a typical presentation of pseudogout. Neuropathic osteoarthropathy (Charcot joint) (C) is the result of denervation of the joint that allows microfractures to occur with daily activities. Radiographs will show severe dislocation, fragmentation, and reactive changes. Rheumatoid arthritis (D) will show subtle demineralization of the bone, fusiform pericapsular swelling, and be associated with joint effusion. Late disease will show fusion of the bones, marginal erosions, and severe narrowing of the joint space. One Step Further Question: What lifestyle modification may help improve the symptoms of osteoarthritis? Answer: Weight loss

Which structure is compressed by a far lateral unilateral disc herniation between L4 and L5 lumbar vertebrae? A L3 nerve root B L4 nerve root C L5 nerve root D No nerve roots as this is below the level of the nerve roots

Correct Answer ( B ) Explanation: Far lateral unilateral disc herniation between L4 and L5 usually compresses the L4 root as it crosses the disc before exiting at the L4 intervertebral foramen. Pain may be localized around the medial side of the leg. Numbness may be present over the anteromedial aspect of the leg. The anterior tibial muscle may be weak, as evidenced by the inability to heel walk. The quadriceps and hip adductor group, both innervated from L2, L3, and L4, also may be weak and, in extended ruptures, atrophic. Reflex testing may reveal a diminished or absent patellar tendon reflex (L2, L3, and L4) or anterior tibial tendon reflex (L4). Sensory testing may show diminished sensibility over the L4 dermatome, the isolated portion of which is the medial leg and the autonomous zone of which is at the level of the medial malleolus. Unilateral disc herniation between L3 and L4 results in compression of the L3 (A) or L4 nerve root. A posterolateral unilateral disc herniation between L4 and L5 results in compression of the L5 root (C). The nerve roots are still present at this level (D). Although the spinal cord terminates near lumbar nerves L1 and L2, the spinal nerves continue as a bundle of nerves called the cauda equina. One Step Further Question: At which level do most lumbar disc herniations occur? Answer: More than 95% of disk herniations occur at L4 or L5

Which of the following describes a grade 3 ankle sprain? A Complete ligamentous rupture with concomitant distal fibular fracture B Complete ligamentous rupture with considerable swelling, pain, and significant laxity C Partial tear with mild laxity and moderate pain, tenderness, and instability D Partial tear without laxity and only mild edema

Correct Answer ( B ) Explanation: Grade 3 sprain is a complete rupture resulting in considerable swelling, increased pain, significant laxity, and often an unstable joint. Ankle sprain involves stretching or tearing of the ligaments of the ankle. There are three grades of ankle sprain as determined by the extent of ligamentous injury. This injury is a common cause of morbidity in the general population, and the ankle is the most commonly injured joint complex among athletes. Patients who play sports experience approximately one ankle sprain for every 1000 person-days of competition. It is estimated that more than 23,000 ankle sprains require medical care in the United States per day. Eighty-five percent of all ankle sprains occur on the lateral aspect of the ankle, involving the anterior talofibular ligament and calcaneofibular ligament. There are only 3 grades of ankle sprains and complete ligamentous rupture with concomitant distal fibular fracture (A) would be a fracture and not a sprain. Grade 2 ankle sprain (C) is a partial tear with mild laxity and moderate pain, tenderness, and instability. Grade 1 (D) is a partial tear without laxity and only mild edema.

A 73-year-old woman presents with right knee pain for the past several months that has been worse over the last three days. It is worse with walking and better with rest. She has not tried anything to alleviate the pain. Physical examination reveals bony enlargement of the knee and joint line tenderness. An X-ray is ordered and shows narrowed joint spaces, and subarticular reactive sclerosis. She denies any injury or previous surgery on the knee. Which of the following is the best initial pharmacologic treatment? A Acetaminophen B Ibuprofen C Prednisone D Tramadol

Correct Answer ( B ) Explanation: Ibuprofen has the lowest side-effect profile with the most significant pain relief and is the initial analgesic of choice for osteoarthritis. In mild disease, topical NSAIDs are recommended prior to the initiation of oral NSAID use. History will generally reveal mechanical pain and physical exam may be relatively benign or may reveal crepitus, joint line tenderness, and bony enlargement. It is the most common joint diseases affecting older adults and can lead to disability. For many patients, adequate monotherapy is enough to control symptoms of pain. Other nonpharmacologic treatments include exercise, weight loss (if necessary), and avoidance of aggravating activities. Acetaminophen (A) and are commonly prescribed for pain associated with osteoarthritis; however, these should not be used initially due to safety concerns and an increased awareness of its negligible and non-clinically significant effects on pain. Prednisone (C) may provide pain relief but has an increased side effect profile and should not be used long-term for osteoarthritis. Tramadol (D) is used as an alternative pain relief medication; however, this interacts with many other medications and should be used with caution in older adults. One Step Further Question: What is the definitive treatment for refractory osteoarthritis? Answer: Surgical intervention

A 47-year-old man presents with low back pain that has been worsening over the past few years. Physical exam shows tenderness over his lumbar spine and sacroiliac joints. Radiographs of the spine are obtained and show ligamentous ossification and syndesmophyte formation about the cervical, thoracic, and lumbar spine, as well as fusion of the sacroiliac joints. Blood tests reveal that he is human leukocyte antigen B27 positive. He has been taking naproxen without improvement of his pain. Which of the following is the most appropriate pharmacologic treatment? A Cyclobenzaprine B Infliximab C Leflunomide D Methotrexate

Correct Answer ( B ) Explanation: Infliximab is a tissue necrosis factor (TNF) blocker that has been shown to improve pain and quality of life in patients with ankylosing spondylitis. These medications are typically recommended for patients who fail nonsteroidal anti-inflammatories drugs and other more conservative therapies. Other tissue necrosis factor blockers that have been proven to improve quality of life and pain are etanercept and adalimumab. Ankylosing spondylitis typically presents with symptoms of low back or hip pain beginning before 40 years of age. It is an inflammatory disease associated with the HLA-B27 genotype. Men are typically more often affected than women. This condition is poorly understood and the mean delay to diagnosis is 7-8 years. Physical exam findings will show limited range of motion of the spine with flexion and extension and tenderness over the sacroiliac joints. Kyphosis may occur with advanced age and the individual may show a stooped posture. The most common distribution of pain is in the lower back and sacroiliac joints. Imaging will confirm the diagnosis with X-rays showing erosions, squaring, sclerosis, and fusion of the sacroiliac joints. The classic "bamboo spine" may be seen on radiographs. Treatment includes nonsteroidal anti-inflammatory drugs and physical therapy to reduce pain. If patients fail conservative measures, tissue necrosis factor blockers are recommended. Mortality is increased in patients with the leading cause of death being secondary to cardiovascular risk factors. Cyclobenzaprine (A) is a muscle relaxant and is not indicated for relief of the pain associated with ankylosing spondylitis. Leflunomide (C) and methotrexate (D) are disease-modifying antirheumatic drugs that have not shown positive outcomes when used in the treatment of ankylosing spondylitis. One Step Further Question: What class of pharmacologic agents have been shown to reduce radiographic progression of ankylosing spondylitis? Answer: Nonsteroidal anti-inflammatory drugs

Which of the following is an indication for plain radiographs of the lumbosacral spine in the setting of low back pain? A Age 45 years B History of prostate cancer C Pain present for three weeks D Pain with radiation to the lateral foot

Correct Answer ( B ) Explanation: Low back pain is a frequent presenting complaint in the Emergency Department and the majority of these patients have uncomplicated musculoskeletal pain. Plain radiographs contribute little to the workup and management of these patients in the absence of findings that would be concerning for possible fracture, malignancy, infection, or cauda equina syndrome (the so-called "red flag" diagnoses). Indications for imaging include age < 18 years and > 50 years, history of fever, weight loss, malignancy (e.g. history of prostate cancer), injection drug use, immunocompromised state, trauma, progressive neurologic deficits, and symptoms lasting > 4-6 weeks. Age 45 years (A) and pain present for three weeks (C) are not indications for plain radiographs unless other concerning findings are present. Pain with radiation to the lateral foot (D) is concerning for possible S1 radiculopathy from disk herniation. Plain radiographs are not helpful in the workup of radiculopathy. One Step Further Question: What factors should be considered in prostate cancer screening? Answer: Patient age, prostate volume, digital rectal examination findings, family history, and patient race.

A 58-year-old man presents with low back pain. Which of the following historical or physical examination findings is most consistent with a diagnosis of spinal stenosis? A Cobb angle of greater than 15 degrees B Decreased symptoms on forward flexion C Pain relief when walking downhill D Positive straight leg test

Correct Answer ( B ) Explanation: Spinal stenosis is a progressive degeneration of disc and facet joints causing a narrowing of the canal and compression of nerve roots. This often leads to neurogenic claudication and radicular symptoms which are often described as unilateral or bilateral buttock pain. In addition to this described pain, spinal stenosis may lead to sensory changes, decreased deep tendon reflexes, and mild weakness. Diagnostic imaging, along which exam findings, are helpful in establishing a diagnosis. X-rays of the lumbar spine may show narrowing of the intervertebral discs or old burst fracture, while an MRI will demonstrate stenosis. Management consists of physical therapy to improve core strength, anti-inflammatory agents, and surgical decompression for progressive disease. It may also be helpful to make certain changes to a patient's posture as specific movements or positions can either aggravate or relieve spinal stenosis symptoms. For patients with spinal stenosis, rest and forward flexion relieve compression of the nerve roots leading to decreased symptoms. The Cobb angle measures the angle from the tilted vertebrae above and below the apex of a given curve in the spine. A Cobb angle of greater than 15 degrees (A) is diagnostic for scoliosis, not spinal stenosis. Patients with spinal stenosis often feel worse when walking downhill (C) because it forces them to extend their backs, placing more compression on their spinal canal and nerve roots. The straight leg test stretches the lower lumbar roots, causing pain for patients with herniated lumbar discs. Therefore, a positive straight leg test (D) indicates a diagnosis of sciatica caused by a herniated lumbar disc, not spinal stenosis. One Step Further Question: What is a common description of the relief felt by spinal stenosis patients when bending forward? Answer: The shopping cart sign.

A 79-year-old woman complains of increasing fatigue and general weakness. She has never fallen in her life before, but, in the past six months, she has had three falls. When considering your differential diagnosis, which of the following age-related body composition changes would you explain to the patient? ABone demineralization BMyofibrosis CNeuroregeneration DPresbycusis

Correct Answer ( B ) Explanation: The aging process can be summarized by describing the "Senescent Phenotype", which includes the four physiological processes of body composition change, neurodegeneration, homeostatic dysregulation and altered energy dynamics. Concerning body composition, a person's weight will change over life, increasing from birth to about 65 years of age, at which point it decreases, mainly due to an overall decrease in protein and fat. However, the ratio of fat to protein increases due to fatty infiltration. One main target of this process is muscle tissue: non-collagenous protein gets replaced with fat. But, the muscle protein that does remain becomes more fibrotic, with an overall increase in fibro-collagenous tissue, a process known as fibrosis. Increased muscle fat and fibrosis, and decreased functional muscle protein, leads to decreased muscle function which manifests as weakness. Bone demineralization (A) does occur with aging, however, it's main effect is an increase in fractures, not a decrease in strength. We are born with all the neurons we'll ever have; i.e., we do not regenerate nerves (C). Presbycusis (D) is the age-related decrease in hearing. One Step Further Question: Although overall fat decreases with age, where does the distribution of fat tend to accumulate? Reveal Answer: Rapid Review Aging Changes ↓ Hearing, vision ↓ Immune response ↓ Bladder control ↓ Pulmonary, renal, GI function ↑ Fat/muscle ratio ↑ Suicide risk Sleep: ↓ REM sleep ↑ Sleep onset latency, ↑ Early awakenings Sexual changes: Males: ↑ refractory period, slower erection/ejaculation Females: vaginal dryness, thinning, shortening

A 60-year-old man presents to the office with a history of osteoporosis. Which of the following statements is correct as it relates to men with osteoporosis? A Anorexia is a common finding B Back pain with vertebral compression is the most common presenting complaintCorrect Answer C Gonadotropin releasing hormone analogues for prostate cancer lessens the chances of developing osteoporosis D Secondary causes includes hyperthyroidism and coronary artery disease

Correct Answer ( B ) Explanation: The clinical presentation of osteoporosis in men is often different from that in women, especially in relation to the time to diagnosis after symptoms begin. Back pain with vertebral compression is the most common presenting complaint. Bone density measurements are less frequently obtained in men, but they are performed after symptoms occur, a far different situation than in postmenopausal women. Lack of androgens results in a skeletal deficit. Peak bone mass is clearly reduced in androgen-insufficient young male patients whether the condition results from idiopathic hypogonadotropic hypogonadism, Klinefelter's syndrome, or constitutional delayed puberty. Anorexia (A) in male patients is extremely rare, and the athlete's triad so characteristic of women runners (i.e., exercise, hypogonadism, and low bone mass) is infrequent in men. The use of long-acting gonadotropin-releasing hormone analogues (C) that block androgen production and are administered in the treatment of prostate cancer is also associated with significant bone loss and fractures in later life. Secondary causes (D) of osteoporosis dominate this disease in men. Hypogonadism and hypercortisolemia are the major etiologic factors in male osteoporosis and must be considered regardless of the phenotypic presentation. However, certain other conditions, including gluten enteropathy, gastric resection or bypass, and ethanol abuse, are more common in men than women with osteoporosis. One Step Further Question: Is estrogen replacement therapy (ERT) indicated in primary prevention of osteoporosis in women? Answer: Although once believed to be beneficial, ERT therapy is no longer indicated for primary prevention of osteoporosis. Rapid Review Osteoporosis Decline in bone mass with aging → ↑ bone fragility + ↑ fracture risk F > M Risk factors: alcohol, steroid use, whites, Asians Height loss Most common fracture: vertebral body compression fractures DEXA scan T-score ≤ -2.5 Prevention: weight-bearing exercises, calcium, vitamin D, smoking cessation Pharmacologic rx: bisphosphonates (1st line)

Which of the following is classically seen in flexor tenosynovitis? A Extended position of the involved digit B Fusiform swelling of the digit C Tenderness over the extensor sheath D Vesicular eruption over the flexor surface

Correct Answer ( B ) Explanation: The flexor tendons of the fingers are covered by a double layer of synovium to promote gliding of the tendon underneath. Infections in the synovial spaces in the hand tend to spread along the course of the flexor tendon sheaths and may extend proximally to the hand. Infections are usually due to penetrating trauma involving the sheath but occasionally from hematogenous spread. Four cardinal signs of acute flexor tenosynovitis are usually present to help distinguish tenosynovitis from other hand infections. These criteria are referred to as the Kanavel's signs. Flexor tenosynovitis is a surgical emergency. Consultation with a hand surgeon is warranted along with intravenous antibiotics. The affected digit is held in a flexed (A), not extended, posture. The tenderness is over the flexor (C) sheath, not extensor. Vesicles (D) are not commonly associated with flexor tenosynovitis. A localized herpes simplex infection may cause vesicles to form on a digit. One Step Further Question: What are complications of flexor tenosynovitis? Answer: Necrosis and proximal spread. Rapid Review Flexor Tenosynovitis Penetrating trauma → S. aureus infection Kanavel's cardinal signs ​Fusiform or symmetrical swelling of finger Flexed posture of finger Tenderness along flexor tendon Pain with passive range of motion ​ABX, surgical emergency

A 34-year-old man presents to the emergency department complaining of extreme left lower leg pain. Three hours earlier, his leg was crushed between his truck and trailer. On physical exam, the pain is exacerbated by passively dorsiflexing the ankle and the leg feels hard and firm. There is diminished sensation in the left foot. Which of the following is the most likely diagnosis? A Acute arterial occlusion B Compartment syndrome C Deep venous thrombosis D Superficial thrombophlebitis

Correct Answer ( B ) Explanation: The man in this case most likely has acute compartment syndrome. Compartment syndrome occurs when tissue pressure within a closed compartment compromises perfusion and results in muscle and nerve ischemia. In the extremities, compartment syndrome may occur acutely, usually following trauma, or chronically, seen usually in athletes. Acute compartment syndrome is much more common than chronic compartment syndrome. Fractures are the most common cause of acute compartment syndrome. Other potential causes include crush injuries, thermal burns, highly constrictive bandages, penetrating injuries, bleeding disorders, animal envenomations, and extravasation of intravascular fluids. The anterior compartment of the lower leg is the most common site. One of the most common and earliest symptoms of ACS is pain out of proportion to the apparent injury. Physical manifestations may include pain with passive muscle stretching, firm "wood-like" feeling of the compartment, pallor from compromised vasculature, diminished sensation, or muscle weakness. Motor deficits are late findings. The diagnosis is based on clinical findings and serial measurements of compartment pressures. Immediate surgical consultation should be obtained if acute compartment syndrome is suspected, as this is a surgical emergency. Initial management involves removing any dressings, splints, or casts and keeping the limb level with the torso. Definitive treatment is a fasciotomy. Complications of untreated acute compartment syndrome include muscle contractures, sensory deficits, paralysis, and infection. Acute arterial occlusion (A) manifests with the 5 P's: pain, pallor, pulselessness, paresthesias, and paralysis. Atrial fibrillation is one of the most common causes of acute arterial occlusion. Based on this man's age and history of acute trauma, acute compartment syndrome is more likely. Deep venous thrombosis (C) occurs when blood coagulates within one of the deep veins. Symptoms of deep vein thrombosis include edema, pain, tenderness, warmth, and erythema. Deep vein thrombosis risk factors include immobility, hypercoagulability, and vascular injury. It is unlikely that the man in this case developed a deep vein thrombosis in the three hours after his initial injury. Superficial thrombophlebitis (D) is caused by thrombus formation in a vein located near the skin. Superficial thrombophlebitis is typically seen in patients who are hypercoagulable or immobilized, in patients with varicose veins or systemic lupus erythematosus, and in patients with venous catheters. Superficial thrombophlebitis typically causes induration, erythema, and tenderness along the affected vein. One Step Further Question: What is the normal pressure of a tissue compartment? Answer: < 10 mmH

A 24-year-old long distance athlete was brought to the emergency room complaining of severe leg pain in his left anterior lower leg. The pain became worse while running his ultra marathon yesterday and subsided after he finished. However he went for a run today and the pain returned. He describes the pain as a burning, tight pain that is 10/10. On physical exam he is in exquisite discomfort. There are no signs of trauma or broken bones. The pain is worsened on passive stretching of the leg. On palpation of his legs there is a firm wooden feeling. Distal pulses are palpated however his left leg does appear very pale. He has diminished 2-point sensory discrimination in his left leg compared to his right leg. Which of the following is the treatment of choice for this condition? A Anticoagulation B FasciotomyCorrect Answer C Hyperbaric oxygen therapy D Narcotic medications for pain relief

Correct Answer ( B ) Explanation: This patient has compartment syndrome of his left anterior leg. Compartment syndrome occurs when the tissue pressure within a closed muscle compartment exceeds the perfusion pressure and results in muscle and nerve ischemia. Compartment syndrome may affect any compartment, including the hand, forearm, upper arm, abdomen, buttock, and entire lower extremity. Almost any injury can cause this syndrome, including injury resulting from vigorous exercise. Patients with compartment syndrome typically present with pain whose severity appears out of proportion to the injury. The pain is often described as burning. The pain is also deep and aching in nature and is worsened by passive stretching of the involved muscles. The patient may describe a tense feeling in the extremity. The traditional 5 P's of acute ischemia in a limb (pain, paresthesia, pallor, pulselessness, poikilothermia) are not clinically reliable; they may manifest only in the late stages of compartment syndrome, by which time extensive and irreversible soft tissue damage may have taken place. Compartment syndrome has been found in soldiers and athletes without any trauma. This can be acute or chronic, with acute compartment pressures as high as those found in severe trauma. Chronic compartment syndrome (CCS) is a recurrent syndrome during exercise or work. CCS is characterized by pain and disability that subside when the precipitating activity is stopped but that return when the activity is resumed. If compartment syndrome is suspected, check intracompartmental pressure, even in the absence of any trauma history. The definitive therapy for compartment syndrome is emergent fasciotomy (compartment release), with subsequent fracture reduction or stabilization and vascular repair, if needed. The goal of decompression is restoration of muscle perfusion within 6 hours. Anticoagulation (A) is appropriate for deep vein thrombosis; however this patient does not have any risk factors for a blood clot nor does his left leg show signs of swelling or tender chords which are common signs of a deep vein thrombosis. Hyperbaric oxygen therapy (B) has been shown to be beneficial before surgery in order to show the demarcation between viable and non-viable tissue, however this treatment is not first-line and many committees advise against it. The patient will need narcotic medications (D) for pain relief but this is only symptom relief and will not be the treatment of choice. The pressure must be relieved from the inner compartments of the leg. One Step Further Question: What is the intracompartmental pressure threshold value to diagnose compartment syndrome? Answer: 30-40 mm Hg. Rapid Review Compartment Syndrome PE will show Paresthesias, Pallor, Pulselessness, Poikilothermia, Paralysis, and Pain out of proportion to exam (6 P's) Most commonly caused by tibia fracture If Delta pressure > 30 mm Hg treatment is fasciotomy Comments: Most common sites - forearm, lower leg. Pain is the first symptom.

A 37-year-old man presents with worsening back pain for 1 week. He states that he has had back pain for years but over the last week, his pain has increased and he has weakness in his legs. Examination reveals 4/5 strength in the left leg and 3/5 strength in the right leg with decreased sensation. He has numbness to his buttocks and a post-void residual of 150 ml. What imaging study should be obtained? A CT of the lumbar spine B MRI of the lumbar spine C Non-contrast head CT D X-ray of the lumbar spine

Correct Answer ( B ) Explanation: This patient presents with signs and symptoms concerning for cauda equina syndrome and should emergently have an MRI done to determine the cause of compression. The syndrome results from compression of multiple lumbar and sacral nerve roots usually caused by a central disk herniation. It can also be caused by trauma, malignancy, epidural hematoma, or abscess. In cauda equina syndrome patients have a sudden onset of back pain and radiculopathies of multiple levels involving both legs. Changes in bowel or bladder function also commonly occur with urinary retention being the most consistent examination finding. As neurogenic bladder develops, patients will develop overflow incontinence. Urinary retention has a high sensitivity (90%) and negative predictive value (99.99%). Thus a low postvoid residual makes the disease highly unlikely, although it can not be used to definitively rule out the diagnosis. Other common findings are saddle anesthesia (75%) and decreased rectal tone (60-80%). MRI is the best modality for determining the cause as it has excellent tissue resolution particularly of the intervertebral disks and spinal cord. MRI also has the ability to detect more subtle soft tissue pathology as seen in epidural abscess. In patients who cannot get an MRI, CT myelogram is a reasonable alternative. CT of the lumbar spine (A) is useful in diagnosing acute fractures but has poor resolution of the spinal cord and intervertebral disks. Non-contrast head CT (C) is not helpful as the patient has a peripheral disorder. X-ray of the lumbar spine (D) has low utility, as it does not adequately visualize the spinal cord, intervertebral disks or other soft tissues. One Step Further Question: What are the common etiologies of spinal subarachnoid hemorrhage? Answer: Arteriovenous malformation, tumors and anticoagulation.

An 18-year-old marathon runner presents with diffuse muscle aches and weakness. The symptoms began after he ran his first marathon three days ago. His urine is dark and has 3+ blood on dipstick with 5 RBCs/hpf on microscopy. Which of the following tests is most likely diagnostic for this disorder? A Non-contrast CT of the abdomen and pelvis B Serum creatine kinase C Serum myoglobin D Serum potassium

Correct Answer ( B ) Explanation: This patient presents with symptoms consistent with rhabdomyolysis, which can be diagnosed by an elevated creatine kinase (CK). Rhabdomyolysis describes the breakdown of skeletal muscle with release of intracellular contents into the blood stream. These contents include creatine kinase, aspartate transaminase, lactate dehydrogenase, myoglobin, aldolase and electrolytes. Muscle breakdown leads to swelling, edema, increased compartment pressure and ischemia. The entry of intracellular components into the bloodstream can lead to renal impairment, insufficiency and irreversible damage. There are four major etiologies of rhabdomyolysis: 1) impaired muscle ATP production, 2) disruption of nutrient delivery to skeletal muscle, 3) increased metabolic demands of skeletal muscle and 4) direct muscle damage. Patients typically complain of swelling and tenderness of muscle groups as well as cramping and pain. Tea-colored urine often prompts physician evaluation. The disease is commonly suspected by the presence of blood on dipstick with the absence or only minimal amount of RBCs on microscopy. The positive dipstick is the result of myoglobinuria. Elevated CK levels are diagnostic of rhabdomyolysis. The enzyme rises over 12 hours and stays elevated for 5 days. Treatment of rhabdomyolysis focuses on identifying compartment syndrome, correcting electrolyte disorders and fluid resuscitation to correct renal impairment if present. A non-contrast CT of the abdomen and pelvis (A) is indicated for the diagnosis of urolithiasis, which presents with hematuria but this patient does not have a clinical scenario consistent with this disease. Serum myoglobin (C) is useful in the early presentation of rhabdomyolysis but it returns to normal within 24 hours. Serum potassium (D) may be elevated in rhabdomyolysis as the result of renal impairment and mycocyte breakdown but is not diagnostic. One Step Further Question: Does the serum CK level predict the risk of acute renal failure? Answer: No. CK level correlates well with degree of muscle injury but does not predict acute renal failure. Rapid Review Rhabdomyolysis RFs: trauma, heat, alcohol/drugs, exercise CPK > 5 times ULN Urine: tea-colored, + for blood but - for RBCs, myoglobinuria Hyperkalemia, hyperphosphatemia, hypocalcemia Immediate ECG Rx: IVF, bicarbonate Complications: DIC, ARF, compartment syndrome

Which of the following stable patients with back pain should have an emergent MRI performed? A 23-year-old man with a positive straight leg raise and otherwise normal neurologic examination B 30-year-old woman with back pain and dysuria C 34-year-old man with a history of intravenous drug abuse with back pain and constipation D 45-year-old woman with back pain after a car accident with no midline tenderness and normal neurologic examination

Correct Answer ( C ) The majority of patients who present with back pain do not require emergent imaging. However, those who may have a serious cause of their back pain including cauda equina syndrome, epidural abscess, vertebral osteomyelitis and other causes of cord impingement should be considered for emergent imaging of the spine. Patients that present with back pain should be screened for "red flags" in their presentations that support one of these critical diagnoses. These red flags include age (both extremes), history of cancer (especially those known to metastasize to bone), fever, weakness, saddle anesthesia, change in bowel or bladder function (incontinence or retention/constipation), trauma, history of immunocompromise (including chronic steroid use) and intravenous drug abuse. Patients with these components to their presentations have a higher risk of dangerous diagnoses and should be considered for early imaging with either CT (better for bony abnormalities) or MRI (better for spinal cord visualization A positive straight leg raise (A) (pain radiating from the back into the leg below the knee) is suggestive of a protruding disk but can be worked up as an outpatient. Patients with symptoms of pyelonephritis (B) rarely require imaging unless there is a more complicated picture to their presentation (presence of kidney stone, sepsis, septic shock). Patients with trauma but no midline tenderness and a normal neurologic examination (D) do not require imaging. One Step Further Question: What critical diagnoses should be considered in patients with syncope and back pain?

Which of the following is not a relative contraindication to an arthrocentesis? A Anticoagulant usage B Bacteremia C History of arthrocentesis D Infection of the overlying skin E Prosthetic joints

Correct Answer ( C ) Explanation: A history of a previous arthrocentesis (C) is not considered a contraindication to joint aspiration. Known bacteremia (B) is considered a relative contraindication as are bleeding diatheses. Anticoagulation (A) use is considered a relative contraindication to arthrocentesis. Infection in the tissues overlying the site to be punctured (D) is generally considered a relative contraindication to arthrocentesis. Prosthetic joints (E) are at high risk for infection and therefore arthrocentesis should be avoided whenever possible. However this is not an absolute contraindication and should be performed if clinically indicated. One Step Further Question: What is the most common source of septic arthritis? Answer: Hematogenous spread. Rapid Review Arthrocentesis Contraindications: overlying skin infection, bleeding diathesis, bacteremia Ankle: medial to the anterior tibial tendon and directed toward the anterior edge of the medial malleolus. Elbow: lateral aspect distal to the lateral epicondyle and directed medially Knee: midpoint or upper portion of patella and directed beneath the posterior surface of patella into joint Shoulder: inferior and lateral to the coracoid process and directed posteriorly toward the glenoid rim

Which of the following is the preferred study to diagnose cauda equina syndrome? A Lumbar puncture B Lumbar spine CT scan C Lumbar spine MRI scan D Lumbar spine X-ray

Correct Answer ( C ) Explanation: A lumbar spine MRI scan is the preferred study to diagnose cauda equina syndrome. Cauda equina syndrome is the compression of the lumbosacral nerve roots below the level of the conus medullaris and is considered a surgical emergency. This disorder can be caused by tumors or metastases, spinal infection, or herniation of a disc causing spinal stenosis. Symptoms include low back pain with radicular symptoms, saddle anesthesia, and bowel or bladder dysfunction. MRI allows for the most accurate visualization of all structures of the spine, including the bones, tissues, and spinal canal. An urgent lumbar spine MRI scan is necessary for patients with a chief complaint of new-onset urinary or bowel dysfunction in the setting of low back pain. A lumbar spine CT scan (B) is not as useful as a lumbar spine MRI in the setting of cauda equina syndrome because it does not demonstrate the spinal canal nor the epidural space as clearly. Because of this, obtaining an MRI scan is superior to a CT scan. In cases where a patient is unable to undergo a lumbar spine MRI, a CT myelogram will suffice because this modality uses contrast dye to assess the spinal canal. A lumbar puncture (A) can be performed at the bedside if an infection is suspected, but it is not a test that should be performed when cauda equina syndrome is suspected. Assessing the patient's vital signs, obtaining a good history from the patient, and reviewing laboratory data and radiography will put an infectious process either higher or lower on the differential diagnosis. If suspicion of an infection is high, then a lumbar puncture should be performed. A lumbar spine X-ray (D) is not useful in the case of cauda equina syndrome because it is unable to visualize the thecal sac. A lumbar spine X-ray can be used to assess the alignment of the lumbar spine and to determine if spinal stenosis is present, but it does not assess the spinal cord well and therefore is an unnecessary test to perform. One Step Further Question: What assessment should be a part of the physical exam when assessing for cauda equina syndrome? Answer: Digital rectal exam to assess rectal tone

A 19-year-old man presents with pain in his index finger. Which of the following is suggestive of flexor tenosynovitis? A Delayed capillary refill B Holding the finger fully extended C Pain on passive extension D Swelling localized to the volar aspect of the finger

Correct Answer ( C ) Explanation: Flexor tenosynovitis is an infection of the flexor tendon sheath of the finger. Most commonly, penetrating trauma to the actual sheath causes this infection although direct spread from other areas of the hand may occur. Staph aureus and streptococci are the most common causative organisms. Kanavel's signs are the four cardinal signs of acute flexor tenosynovitis include: 1) tenderness along the flexor tendon, 2) symmetric swelling of the finger ("sausage finger"), 3) pain on passive extension and 4) flexed posture of the finger. These patients need antibiotic therapy and admission to the hospital for possible operative drainage. Delayed capillary refill (A) may be seen with significant swelling in a finger infection but is not a hallmark sign for flexor tenosynovitis. Typically the pain of the infection is exacerbated when the finger is fully extended and as a result patients avoid holding the finger fully extended (B). More often, the finger will be flexed and on passive extension by the clinician the pain is exacerbated. Swelling is not localized to the volar aspect of the finger (D) but is more symmetric and uniform along the entire finger creating the sausage-like appearance. One Step Further Question: What is a felon? Answer: An infection of the pulp of the distal finger or thumb. Rapid Review Flexor Tenosynovitis Penetrating trauma → S. aureus infection Kanavel's cardinal signs ​Fusiform or symmetrical swelling of finger Flexed posture of finger Tenderness along flexor tendon Pain with passive range of motion ​ABX, surgical emergency

Which of the following diagnostic studies has the highest sensitivity to evaluate for acute radiculopathy? A Cerebrospinal fluid analysis B Electromyography C Magnetic resonance imaging D X-ray

Correct Answer ( C ) Explanation: Magnetic resonance imaging (MRI) should be performed when evaluating for acute radiculopathy because it is the most sensitive diagnostic tool for a herniated nucleus pulposus or other intraspinal pathology. Radiculopathy is a disease of a nerve root caused by compression, irritation, or swelling. It most commonly is a result of a herniated disc but can also be due to malignancy, infection, or vascular disorders. An MRI is the diagnostic test of choice because it is the least invasive and least radiation-inducing procedure to visualize the cause of radiculopathy. If a patient can't undergo an MRI because of intolerance or a contraindication, a CT scan or CT myelogram with contrast can be performed instead. Cerebrospinal fluid analysis (A) is not routinely performed in the workup of radiculopathy and is only indicated if no other causes are identified or if there is a high suspicion for neoplasm or infection. Leptomeningeal carcinomatosis, Lyme disease, and cytomegalovirus are examples of causes of radiculopathy that may only be found through testing of the cerebrospinal fluid. A lumbar puncture, however, should not be obtained when there is high suspicion for an epidural abscess because there is a high risk of introducing infection into the central nervous system if the needle passes through an infected area. Electromyography (B), also known as EMG, is used to evaluate the electrical activity produced by muscles and the nerve roots that control them. This study examines the integrity and utilization of nerve roots and can help to localize injured nerves as well as provide information as to how long the nerve has been injured. An EMG is not routinely performed and should be reserved for those whose neuroimaging findings do not correlate with the patient's symptoms or when it is important to determine how long the nerve has been damaged. X-ray (D) is not routinely performed when evaluating radiculopathy as it can only visualize bony processes and not the discs and therefore would not provide enough information to make a diagnosis. If fracture, spinal stenosis, infection, or malignancy are suspected, an X-ray should be performed to evaluate the alignment of the spine, but it should not routinely be performed. One Step Further Question: In CSF analysis, what does xanthochromia indicate? Answer: Red blood cells in the subarachnoid space

A 67-year-old woman has a bone mineral density T-score of -2.8 standard deviations. Which of the following is the most likely diagnosis? A Normal bone density B Osteopenia C Osteoporosis D Severe osteoporosis

Correct Answer ( C ) Explanation: Osteoporosis is a very common disease that leads to increased risk of bone fracture in those affected. Risk factors for osteoporosis include advanced age, female gender, Caucasian and Asian race, body weight less than 58 kg, cigarette smoking, increased alcohol intake, previous fracture and family history of hip fracture. Bone mineral density (BMD) is used to assess risk and is determined by using dual-energy x-ray absorptiometry (DXA) scan. DXA scan results are reported as T-scores which provide information about the progression of disease and risk of fracture, categorizing patients on the spectrum from normal bone density to severe osteoporosis. Osteoporosis is defined by a BMD of 2.5 or more standard deviations below the young adult female reference mean which is equivalent to a T-score of less than or equal to -2.5 standard deviations. Normal bone density (A) is defined as a T-score greater than or equal to -1 standard deviations. Osteopenia (B), also known as low bone mass, is diagnosed with a T-score of between -1 and -2.5 standard deviations. Severe osteoporosis (D) occurs with the presence of one or more fractures and a T-score of less than or equal to -3.5 standard deviations. One Step Further Question: Which part of the body is scanned to determine bone mineral density? Answer: Hip and spine

An 82-year-old woman with osteoporosis falls against her kitchen table. She presents with acute right shoulder pain, proximal right upper extremity edema and near-absent active shoulder joint range of motion. Which of the following is the most appropriate next step in management? AObtain a scapular-Y radiograph Your Answer BPassive range of motion measurement CPerform a distal neurovascular examination DPerform a Hawkins testing

Correct Answer ( C ) Explanation: Proximal humerus fractures are most commonly seen in elderly patients. The most common type of fracture occurs at the surgical neck (the epiphyseal-diaphyseal junction, just inferior to both tuberosities). As with any fracture, it is imperative to assess distal neurovascular supply with motor, sensory, and pulse examination. Complications from shoulder injuries include injury to the brachial plexus and vascular such as the axillary artery. The Hawkins test (D), which usually follows Neer's testing of the shoulder, is used to evaluate subacromial impingement in the setting of rotator cuff syndrome or tear, the results of which are most likely noncontributory given this presentation. Passive range of motion measurements (B) will most likely be noncontributory and difficult to obtain. Radiographs (A) occur after the clinician is assured that no acute neurovascular compromise is present. One Step Further Question: The greater tuberosity is the insertion point of which 3 rotator cuff muscles? Answer: Supraspinatus, infraspinatus and teres minor. Rapid Review Proximal Humerus Fracture Neer classification: based on number of displaced (> 1 cm) or angulated (> 45°) parts Most common fracture: one part fracture Orthopedic consultation for displaced fractures

Which of the following is a risk factor for an Achilles tendon rupture? A Age older than 60 years B Diabetes C Fluoroquinolone usage D Steroid taper

Correct Answer ( C ) Explanation: Rupture of the Achilles tendon is relatively common. It is most common in men between the ages 30 and 50 years who participate in recreational sports—the weekend warrior. However, it can occur in serious athletes. Exogenous risk factors include chronic corticosteroid usage and fluoroquinolone usage. Age older than 60 years (A) is not a risk factor. Most cases occur in patients between the ages of 30 and 50 years. Diabetes (B) is not a risk factor for Achilles tendon rupture. Chronic, rather than short-term, steroid usage (D) predisposes to Achilles tendon rupture. One Step Further Question: What is the classic maneuver to test the integrity of the Achilles tendon? Answer: The Thompson test: Patient is prone and knee at 90 degrees. Squeezing the calf muscle should cause plantar flexion of the foot with an intact Achilles tendon. Rapid Review: Achilles Tendon Rupture Patient will be a deconditioned athlete With a history of fluoroquinolone use Complaining of "pop" or "snap" and sudden pain in the calf area PE will show absent plantarflexion upon calf squeeze (Thompson test) Treatment is posterior splint in plantarflexion, orthopedic consult

Which of the following is necessary to confirm the diagnosis of scoliosis? A Adam's forward bend test B Magnetic resonance imaging C Radiography D Scoliometer

Correct Answer ( C ) Explanation: Scoliosis is a lateral curvature of the spine. Idiopathic scoliosis is the most common type of scoliosis and within this category, the most common type is adolescent idiopathic scoliosis (AIS). AIS is defined as a Cobb angle greater than or equal to 10 degrees, age of onset 10 years or greater and no identifiable etiology. The Cobb angle is determined on radiography and is the measurement used to monitor the progression of scoliosis. Risk factors for the development of AIS include age less than 12 years, onset prior to menarche, curves with a Cobb angle greater than or equal to 20 degrees, female sex, and double or thoracic curves. The curvature of scoliosis can progress during periods of rapid growth, such as adolescence. This can cause significant deformity and other clinical manifestations such as cardiopulmonary compromise, therefore it is important to identify and monitor patients with this condition. A number of screening measures are used to determine the likelihood of scoliosis, but radiography is needed for the diagnosis of scoliosis as determined by the Cobb angle. Physical examination for scoliosis includes the Adams forward bend test (A). The patient is observed from the back while bending forward at the waist until the spine is parallel to the ground. Lumbar or thoracic prominence on one side indicates possible scoliosis and further workup should be initiated. Magnetic resonance imaging (B) is indicated when plain radiography is suggestive of an intraspinal pathology, such as a tumor or infection. Scoliosis can be diagnosed on X-ray and MRI is unnecessary. A scoliometer (D) is a tool used for scoliosis screening and can quantify the amount of trunk rotation. Accurate use of the scoliometer is based on operator experience. Question: Which abnormal reflex is most indicative of intraspinal pathology? Answer: Abdominal

An 18-year-old girl presents to the ED with left ankle pain. Earlier in the day she was playing softball and slid into second base and "twisted her ankle." On exam, you note moderate swelling, tenderness, and pain with passive range of motion of the ankle. You do note some abnormal motion when stressing the joint. Which of the following is the most likely diagnosis? A First-degree sprain B First-degree strain C Second-degree sprain D Second-degree strain

Correct Answer ( C ) Explanation: Sprains are classified as ligamentous injuries resulting from an abnormal motion of a joint. In such cases, there is injury to the ligamentous fibers of a supporting joint. Sprains are graded according to the severity of pathologic findings; however, clinically the grades are often indistinct. A second-degree sprain is a partial tear of a ligament (more than first-degree). Clinically, there will be moderate hemorrhage and swelling, tenderness, painful motion, abnormal motion, and loss of function. Although there may be some laxity with stressing of the joint, an absence of end points will be seen only with complete ligament rupture (i.e., third-degree sprains). A first-degree sprain (A) is characterized by minor tearing of ligamentous fibers with mild hemorrhage and swelling. There is minimal point tenderness. Stressing the ligament produces some pain, but there is no opening or abnormal joint motion. A strain is an injury to a musculotendinous unit resulting from violent contraction or excessive forcible stretch. Sometimes the term "pulled muscle" is used interchangeably with muscle strain. A first-degree strain (B) is a minor tearing of the musculotendinous unit, characterized by swelling, local tenderness, and minor loss of function. With a second-degree strain (D), more fibers are torn but without complete disruption. There is also greater swelling, ecchymosis, and loss of strength. One Step Further Question: What is the traditional treatment for sprains? Answer: Ice, elevation, and analgesia with NSAIDs. Immobilization is used to provide protection and comfort in the initial management.

A 14-year-old boy presents to clinic with a one month history of right shoulder pain. The patient is a competitive tennis player. Symptoms began during a match when the patient was serving. He experienced a pop in the shoulder and had immediate pain. Magnetic resonance imaging of the shoulder shows a superior labral tear from anterior to posterior (SLAP lesion). Which of the following is the most likely physical exam finding A Positive cross arm test B Positive Neer test C Positive O'Brien test D Positive sulcus test

Correct Answer ( C ) Explanation: The O'Brien test (also known as the active compression test) is performed with the patient standing. The patient holds his shoulder in 90 degrees of forward flexion with the elbow extended. The arm is then adducted 10 degrees. The patient is instructed to internally rotate his arm, pointing his thumb to the floor. The patient should hold his arm in this position against resistance while the examiner pushes the arm towards the floor. The examiner then applies the same force with the patient's arm externally rotated. The test is considered positive if the patient has pain or popping in the internally rotated position and the pain is eliminated in the externally rotated position. The test is thought to be sensitive for a SLAP (superior labral tear from anterior to posterior) lesion if the patient reports deep, diffuse glenohumeral joint pain. SLAP lesions can be secondary to an acute injury or chronic overuse, such as in overhead athletes. Patients may complain of clicking or mechanical symptoms, especially when the arm is the in cocked position of throwing or serving. The cross arm test (A) is used to asses for acromioclavicular joint pain. In this test, the patient raises his or her arm to 90 degrees and then actively adducts the arm. The test is positive if this elicits pain over the acromioclavicular joint. To perform the Neer test (B), the patient should fully pronate the affected arm. The examiner then stabilizes the patient's scapula with one hand while using the other hand to lift the patient's arm into maximal forward flexion. Pain or facial grimacing during this maneuver suggests impingement of the supraspinatus tendon. The sulcus test (D) is performed with the patient's arm in a relaxed neutral position. The examiner pulls the patient's hand downward at the elbow joint. The examiner looks for depression (sulcus) between the lateral acromion and the head of the humerus, suggesting inferior glenohumeral instability. One Step Further Question: What is the treatment for a SLAP tear? Answer: First-line treatment is usually non-operative management with nonsteroidal anti-inflammatory drugs and physical therapy, with consideration for surgery if the patient fails conservative treatment

What is the most appropriate first line treatment for plantar fasciitis? A Corticosteroid injection B Extracorpeal shock wave ultrasound C NSAIDs D Surgical therapy

Correct Answer ( C ) Explanation: The plantar fascia is a tough layer of the sole that is functionally significant during foot strike and the early stance phase of walking. Plantar fasciitis is an overuse injury of insidious onset that usually begins with pain on first weight bearing in the morning or after prolonged sitting. This progresses to persistent pain during gait. Pain and tenderness are localized to the medial aspect of the heel. Plain radiography is not diagnostic but shows a calcaneal spur in 50% of patients with plantar fasciitis. NSAIDs are considered first line treatment, along with rest, ice, and shoe inserts. Corticosteroid injection (A) is considered if conservative therapy fails. Pain relief has only been demonstrated at 1 month with no significant difference at 6 months. Extracorpeal shock wave ultrasound (B) is a second-line therapy with mixed results for any positive benefit. Surgical therapy (D) is considered if all other measures fail. There is no proven benefit that surgical therapy is any more beneficial than conservative therapy. One Step Further Question: Which type of foot anomaly is plantar fasciitis common in? Answer: Cavus feet.

A 48-year-old man presents to the office with right shoulder pain that has been worse over the last 3 months. He has positive Hawkins and Neer signs. There is visible atrophy at the superior and posterior aspects of the shoulder. Which of the following is the most likely diagnosis? ACervical nerve impingement with radiculopathy BGlenohumeral osteoarthritis CRotator cuff impingement DSuprascapular nerve entrapment

Correct Answer ( C ) Explanation: The rotator cuff is made up of the supraspinatus (abduction), infraspinatus (external rotation), teres minor (external rotation), and subscapularis (internal rotation) muscles. Rotator cuff impingement and tearing usually begin in the supraspinatus tendon as it passes under the acromion. Patients typically have pain with abduction above the head and internal rotation (reaching up the back). It is associated with a positive Hawkins (with the patient's elbow and shoulder flexed to 90 degrees the examiner passively internally rotates the shoulder by stabilizing the elbow and pushing down on the wrist) and Neer (the examiner performs maximal passive forward flexion with internal rotation while stabilizing the patient's scapula with the other hand) tests. Cervical nerve impingement with radiculopathy (A) could have caused muscular atrophy, but would not explain signs of rotator cuff impingement. Cervical radiculopathy is usually identified by neck stiffness, deltoid weakness, absent biceps reflex and possible sensory loss. Glenohumeral osteoarthritis (B) is a common cause of shoulder pain with any movement but would not explain rotator cuff impingement. Glenohumeral osteoarthritis also would not cause focal muscle atrophy, and is typically visible on radiograph. Suprascapular nerve entrapment (D) can cause atrophy of the supraspinatus and infraspinatus muscles, but would not have signs of impingement. One Step Further Question: What is the effect of subacromial injection of local anesthesia on a rotator cuff tear? Answer: The strength of the supraspinatus does not improve after injection of local anesthesia.

A 13-year-old boy presents to clinic for evaluation of flat feet. He does not complain of pain. On physical exam, the patient has bilateral pes planus when standing. He is able to recreate an arch when he stands on his toes and has normal subtalar motion bilaterally. Which of the following is the most appropriate course of action? A Arch supports to stimulate arch development B Calcaneal osteotomy to improve foot alignment C Counsel the family that painless flat feet is a normal finding and does not require treatment. D Resection of the tarsal coalition that is causing flat feet

Correct Answer ( C ) Explanation: This patient has flexible flat feet characterized by collapse of medial longitudinal arch and hindfoot valgus when standing. Approximately 20-25% of the population has flat feet. It is a normal physiologic variant and does not typically cause pain or functional limitations. It is important, however, to differentiate flexible flat feet from a rigid flatfoot. A rigid flatfoot is caused by a congenital tarsal coalition, an osseous, fibrous, or cartilaginous bar between the talus and calcaneus or calcaneus and navicular bones. In a rigid flatfoot, the patient has decreased subtalar motion and the collapsed arch does not reconstitute with toe standing. Most cases of rigid flatfoot are asymptomatic but may result in subtalar or midtarsal pain or increased ankle sprains due to decreased foot motion. Patients found to have asymptomatic flexible flat feet do not require any specific treatment other than patient and parental education. Arch supports (A) should be considered in a patient who has pain related to flat feet which may present as arch pain or pretibial pain. Arch supports can help relieve the symptoms but do not change the natural history of flat feet. Calcaneal osteotomy (B) corrects the hindfoot valgus component of flat feet but should only be considered in a skeletally mature patient with painful flat feet that is resistant to extensive nonoperative management including use of orthotics and heel cord stretching. Resection of tarsal coalition (D) with graft interposition should be considered in a patient with a rigid flatfoot secondary to a calcaneonavicular or talocalcaneal coalition. However, this diagnosis is unlikely in a patient with normal subtalar motion and the ability to recreate an arch with toe walking. One Step Further Question: What X-ray findings would suggest a rigid flatfoot secondary to a tarsal coalition? Answer: Radiographs of the foot may show a bony bridge between the calcaneus and navicular or calcaneus and talus

A 6-year-old boy presents with ankle pain after getting kicked while playing soccer. He has tenderness anteriorly along the ankle joint. An X-ray is shown above. The patient has significant pain when walking. Which of the following is the most appropriate management? A Analgesics and follow up in 2 weeks B CT scan of the ankle C Immobilization of the ankle and non-weight bearing D Immobilization of the ankle and weight bearing as tolerated

Correct Answer ( C ) Explanation: This patient presents with signs and symptoms concerning for a Salter-Harris Type I fracture and should be immobilized and made non-weight bearing. Children's bones, unlike those of adults, contain cartilaginous centers near the end of the bone that give rise to new bone growth (epiphysis). Because these areas are radiolucent, they are not visualized on radiographs. Injuries to the epiphysis may result in abnormal bone growth if they do not heal. Therefore, when injury to this area is suspected, conservative management with splinting and non-weight bearing (if lower extremity) status is recommended to promote healing and discourage worsening injury. Injuries to these areas are referred to as Salter-Harris Type I injuries (displacement of the epiphysis). The Salter-Harris classification system was designed to aid in the description and management of pediatric fractures. Analgesics (A) may be required but the patient should be made non-weight bearing and splinted prior to discharge. CT scan of the ankle (B) is not necessary as it would not change treatment at this point. Immobilization is vital but the patient should not be allowed to place weight on the extremity (D). One Step Further Question: In what age group are physeal injuries most common? Answer: Boys aged 12-15 and girls aged 9-12. Rapid Review Salter-Harris Fractures I: S (Slipped epiphysis) II: A (fracture Above physis), most common III: L (fracture beLow physis) IV: T (fracture Through physis) V: R (wRecked physis) I/II rx: nonoperative IV/V rx: surgery required Negative radiographs do not r/o a Salter I fracture

A 35-year-old man presents with a firm, well-circumcised mass on the dorsal aspect of his right wrist. Which of the following physical findings is most consistent with a ganglion cyst? AErythema and warmth BNumbness along C7 nerve distribution CPositive Spurling test DTransillumination

Correct Answer ( D ) Explanation: A ganglion cyst is a benign soft tissue tumor that may affect any joint, but is most often seen in the dorsal aspect of the wrist. The exact pathophysiology is unknown, but it is thought that extra-articular fluid concentrates and is then surrounded by a cyst wall and pedicle that connects the cyst to the nearby synovial joint. Often, ganglion cysts are managed best with watchful waiting as half will spontaneously resolve. For patients who desire removal, aspiration of the cyst and surgical removal are options. However, these treatments have proven to be difficult as there is a high recurrence rate, and surgical removal opens up the possibility for complications, such as infection. Most ganglion cysts are painless, though some patients report pain on activity which is thought to be due to the compression of the cyst on nearby nerves. Physical exam, will reveal a one to two centimeter cystic structure that transilluminates and feels similar to a firm rubber ball. Erythema and warmth (A) are common findings in infectious diseases of the skin and joints, but not in ganglion cysts. Carpal tunnel syndrome is the consequence of the compression of the median nerve, often leading to numbness along C7 nerve distribution (B), which is not a common symptom of ganglion cysts. A positive Spurling test (C) is indicative of cervical radiculopathy, not the presence of a ganglion cyst. The test is performed by turning the patient's head to their affected side while the examiner applies downward pressure. A positive test is if this maneuver reproduces their radicular symptoms. One Step Further Question: Why are ganglion cysts occasionally referred to as Bible cysts? Answer: Ganglion cysts were previously managed with closed rupture (i.e. hitting them with a heavy book, such as the Bible). Rapid Review Ganglion Cyst Patient will be complaining of a painful mass in wrist Most commonly caused by repetitive activity causing tear or degeneration in joint capsule or tendon synovial sheath Treatment is observation or needle aspiration Comments: most common soft tissue tumors of the hand

A 34-year-old woman presents to a local urgent care clinic with insidious onset of right foot pain that has been getting worse over the past two weeks. She denies any known trauma to the foot, but mentions she is training for a marathon. On physical exam, she is moderately tender to palpation over the second right metatarsal. An X-ray of the foot is normal. What is the most likely diagnosis? A Compartment syndrome B Ligament sprain C Osteomyelitis D Stress fracture

Correct Answer ( D ) Explanation: A stress fracture is an overuse injury that is caused by repetitive stress. It is most commonly seen in patients with a history of increased exercise (e.g. marathon training) or a history of increased load (e.g. carrying a heavy backpack). Stress fractures occur when the bone breaks after being subjected to repeated stresses, none of which would be large enough individually to cause the bone to fracture. Risk factors for developing a stress fracture include repetitive activities, decreased physical fitness, inadequate vitamin D and calcium intake, eating disorders, female gender, increased age, decreased bone density, and low body mass index. Radiographs are usually the first line imaging study because they can usually be performed in-office and are cost-effective. However, radiographs have poor sensitivity and are usually normal for the first two to three weeks after injury onset. Periosteal elevation, cortical thickening, sclerosis, and true fracture lines are findings indicative of a stress fracture. Repetitive activities predispose to site-specific stress fractures. For example, a baseball pitcher has an increased risk of humeral and scapular fractures and a runner has an increased risk for pelvic and tibial fractures. The lower extremities account for almost half of all stress fractures. Stress fractures usually cause insidious onset of localized pain that increases with activity. Tenderness to palpation over the affected area is the most sensitive physical finding. Soft tissue swelling may or may not be present, depending on amount of soft tissue overlying the bone. Conservative treatment with activity modification, pain control, and nutrition supplementation, if needed, is preferred over surgical management. Slowing increasing amount of activity helps prevent stress fractures. Compartment syndrome (A), can be seen in runners, but is most commonly found in the lower leg. Typical finding of compartment syndrome are the "5 P's" of pain, pallor, paresthesias, pulselessness, and poikilothermia. Compartment syndrome is more likely to occur after a crush injury or application of a tight cast. A ligament sprain (B) often has an exciting event followed by acute onset of pain. Stressing of the affected ligament would reveal laxity. Osteomyelitis (C) often presents with localized pain and tenderness, but would be unlikely in this case due to lack of contiguous soft tissue infection. Hematogenous seeding in adults typically affects the vertebrae. Clinical suspicion and additional laboratory and imaging studies must be used to rule out osteomyelitis. One Step Further Question: Which sports are considered to be low-risk for developing stress fractures? Answer: Swimming, golf, and softball.

A 31-year-old previously healthy woman presents to your office with a complaint of low back pain. One week ago she was lifting heavy boxes at work when she felt her back "give out". Since then she has had constant pain and decreased range of motion. She has had to take time off work. Which of the following is the most appropriate next step in management? A Bed rest B ExerciseYour Answer C Lumbar support D Patient education

Correct Answer ( D ) Explanation: Acute low back pain is defined as back pain lasting less than 12 weeks and is one of the most common reasons that individuals seek medical treatment in the United States. Approximately 85% of adults will experience at least one episode of acute low back pain in their lifetime. The majority of cases of acute low back pain are due to either musculoligamentous injury or degenerative changes of the spine and most patients improve within 2-4 weeks after the onset of injury, even without intervention. Diagnosis is based on a careful history and physical exam. Imagining is reserved for patients with red flag symptoms such as interruption in bladder or bowel function, significant trauma, fever, history of intravenous drug use or cancer, unexplained weight loss, and pain that is increased or unrelieved by rest. A reasonable approach to patients with acute low back pain on the first visit includes patient education and reassurance, an initial trial of acetaminophen or non-steroidal anti-inflammatory agents, consideration of a short course of muscle relaxers and referral to physical therapy in cases where the patient has a history of back pain or injury. Bed rest (A) is not advised for patients presenting with acute low back pain and may cause adverse effects such as muscle wasting, joint stiffness, pressure ulcers, loss of bone mineral density, and venous thromboembolism. While patients should be encouraged to stay as active as possible, recommending specific types of exercise (B) such as strengthening, flexibility and aerobic exercise has not been shown to be effective in the treatment of acute low back pain. There is no evidence to support the recommendation of lumbar support (C) in treating acute low back pain. Some patients may feel relief of symptoms with lumbar support and it is not likely to cause harm, but should not be included in the initial recommendations for intervention. One Step Further Question: True or false: The majority of cases of acute low back pain will resolve without intervention? Answer: True.

A 19-year old man presents to the ED after rolling his left ankle playing basketball earlier in the evening. X-rays of the ankle do not reveal a fracture. The exam reveals a positive anterior drawer test with no endpoint on the left. Which of the following is the most likely diagnosis? A Achilles' tendon rupture B Grade 1 tear of the calcaneofibular ligament C Grade 2 tear of the deltoid ligament D Grade 3 tear of the anterior talofibular ligament

Correct Answer ( D ) Explanation: Based on the physical exam findings, a grade 3 tear of the anterior talofibular ligament (ATF) is the most likely diagnosis as the anterior drawer test is specific for the ATF and the lack of endpoint indicates a grade 3 or complete rupture of the ligament. The ATF is the most commonly injured ligament in an inversion sprain and is often the ligament that tears first. Inversion ankle sprains are classified by grade with grade 1 being the least severe and grade three being the most severe generally resulting in instability of the ankle and inability to bear weight. In addition to malleolar tenderness, the inability to bear weight if both at the time of injury and in the emergency department is an indication for X-ray via the Ottawa ankle rules. The physical exam of a patient with a ruptured Achilles' tendon (A) would have a positive Thompson's test in which the toes will not plantarflex when the calf muscles are squeezed. An injured deltoid ligament (C) is unlikely due to the mechanism of injury as deltoid ligament injuries generally occur from eversion of the ankle, not inversion. There would be valgus instability of the ankle with a deltoid ligament injury as well. A grade 1 tear (B) of the calcaneofibular ligament (CFL) would result in tenderness and mild swelling about the CFL about its origin on the anterior lateral malleolus to its insertion on the calcaneous. This is the second most commonly injured ligament, after the ATF, with an ankle sprain and would likely be injured with the degree of sprain above. It is evaluated with the talar tilt test with the foot in dorsiflexion and is severely injured if the ankle has increased inversion when the test is performed. One Step Further Question: If there is severe tenderness and swelling at the medial malleolus, which fracture must be suspected? Answer: Maisonneuve fracture defined by fracture of the medial malleolus, disruption of the syndesmosis and fracture of the proximal fibula. Rapid Review Ankle Sprain Patient with a history of ankle inversion PE will show pain and swelling Imaging will show partial or complete tearing of ligaments Most commonly injured anterior talofibular ligament (ATFL) Treatment is RICE therapy Comments: Ottawa Rules to determine imaging

What is the most common cause of acute compartment syndrome? A Femur fracture B Patellar fracture C Scaphoid fracture D Tibia fracture

Correct Answer ( D ) Explanation: Compartment syndrome occurs when there is elevated pressure within a muscular compartment that eventually compromises the circulation and function of the tissues within that compartment. The most common cause of acute compartment syndrome is a tibia fracture. When a fracture occurs, edema accumulates around the area. The tibia is encased within a fascial structure that is unable to expand adequately to meet the demands of increased amount of fluid. Because of this restriction, the amount of fluid within that compartment rises and venous outflow is reduced. A continuous rise in pressure leads to tissue ischemia and therefore tissue loss. While there are four compartments in the lower leg, the anterior compartment is the most common site for acute compartment syndrome, and it is most commonly caused by a tibia fracture. A femur fracture (A) is a fracture of the long bone of the upper thigh. While the thigh does have three compartments, acute compartment syndrome rarely develops in this area unless the injury is caused by a high-impact event. A patellar fracture (B) is a fracture of the patella, or kneecap. Compartment syndrome can develop in this area, but it is very unlikely due to the ability of the fascia in this region to expand. A scaphoid fracture (C) is a fracture of the scaphoid bone in the wrist. The fascia in this region, like in the region of the knee, are more easily expandable, and therefore, developing acute compartment syndrome here is less likely. One Step Further Question: What are the "6 P's" of acute arterial occlusion? Answer: Pain, pallor, pulselessness, paresthesias, paralysis and poikilothermia.

A 56-year-old man has pain and tingling in the medial aspect of his ankle and the plantar aspect of his foot. He jogs 3 miles daily and has no history of any injury. The symptoms are aggravated by activity and sometimes keep him awake at night. The only findings on examination are paresthesias when a reflex hammer is used to tap just inferior to the medial malleolus. Which of the following is the most likely diagnosis? A Diabetic neuropathy B Plantar fasciitis C Stress fracture D Tarsal tunnel syndrome

Correct Answer ( D ) Explanation: Entrapment of the posterior tibial nerve or its branches as the nerve courses behind the medial malleolus results in a neuritis known as tarsal tunnel syndrome. Causes of compression within the tarsal tunnel include varices of the posterior tibial vein, tenosynovitis of the flexor tendon, structural alteration of the tunnel secondary to trauma, and direct compression of the nerve. Pronation of the foot causes pain and paresthesias in the medial aspect of the ankle and heel. Electromyography and nerve conduction velocity studies may be a useful initial tool in evaluating suspected cases of tarsal tunnel syndrome and in confirming the presence of neuropathy. Medical therapy for tarsal tunnel syndrome may consist of local injection of steroids into the tarsal canal. Physical therapy may be of some value in reducing local soft-tissue edema, thereby easing pressure on the compartment. When conservative therapy fails to alleviate the patient's symptoms, surgical intervention may be warranted. The usual site for a stress fracture (C) is the shaft of the second, third, or fourth metatarsals. Plantar fasciitis (B) is the most common cause of heel pain in runners and often presents with pain at the beginning of the workout. The pain decreases during running only to recur afterward. Diabetic neuropathy (A) is usually bilateral and often produces paresthesias and burning at night, with absent or decreased deep tendon reflexes. One Step Further Question: Tarsal tunnel syndrome is analogous to which other nerve entrapment pathology? Answer: Carpel tunnel syndrome

A young woman presents with ankle pain and edema. While wearing high-heel shoes, she twisted her ankle upon stepping off a curb. She is tender about the lateral malleolus. Skin and neurovascular examination are normal. She has no medial tenderness. Ligament testing is negative. Radiographic examination reveals a non-displaced lateral malleolar fracture below the ankle joint. The tibia is unaffected. Which of the following is the most appropriate definitive treatment for this patient? A Closed reduction B Debridement C Non-weight-bearing orthosis D Weight-bearing cast

Correct Answer ( D ) Explanation: Fracture of the ankle may include injury to the medial malleolus (tibia), the lateral malleolus (fibula), the posterior malleolus (tibia), the talus and the collateral ligaments. Stability of the fracture depends on how many sides are injured. Stable fractures involve only one side of the joint, whereas unstable fractures include both sides of the joint. *Stable fractures are treated with 4-6 weeks of a weight-bearing cast or brace.* Unstable, displaced fractures require open or closed reduction (A). Open fractures require surgical debridement (B). Her skin is intact. Unstable, nondisplaced fractures require a non-weight-bearing cast (C). Since she has no radiographic or physical evidence of medial joint involvement, this ankle fracture could be considered stable. Even if you decide this is unstable, a non-weight-bearing cast, not brace (orthosis), is the treatment of choice

A 33-year-old man presents with pain and swelling in his left knee since yesterday. He has also had a fever with a maximum temperature of 102.3°F. Physical exam reveals swelling of the left knee with erythema and warmth noted. There is pain with passive range of motion. What is the most likely diagnosis? A Bursitis B Gout C Osteoarthritis D Septic arthritis

Correct Answer ( D ) Explanation: Septic arthritis is an infection of the joint that is typically characterized by joint pain, joint swelling, and a fever. The involved joint can present with an effusion and be painful with both active and passive range of motion. It is more common in individuals with a history of intravenous drug use, rheumatoid arthritis, immunocompromise, diabetes mellitus, recent joint surgery, or a prosthetic joint. Rigors may be present as well. Joint aspiration and synovial fluid analysis is the mainstay of diagnosis. Serum erythrocyte sedimentation rate (ESR) will be elevated. Management includes hospital admission for parenteral antibiotics directed at Staphylococcal and Streptococcal species including resistant strains of these bacteria. This typically consists of intravenous vancomycin and a third-generation cephalosporin. An emergent orthopedic consultation should be obtained for consideration of surgical irrigation of the joint. Bursitis (A) is an inflammation of the bursa, but the joint is typically not tender or erythematous and fever is not present unless the bursitis becomes infected. Gout (B) will cause erythema and swelling; however, this will not be associated with a fever. Osteoarthritis (C) may cause swelling and pain in the knee joint, but should not be associated with fever or erythema of the joint. One Step Further Question: Which two organisms are likely to cause septic arthritis in two or more joints simultaneously? Answer: Staphylococcus aureus and Streptococcus pneumoniae

A 10-year-old boy presents to clinic with complaints of right posterior heel pain. There was no preceding injury. The pain has been ongoing for four months. He is a soccer player and notices the pain primarily when wearing his cleats during soccer. On physical exam, the patient has pain with medial-lateral compression of the calcaneus. Which of the following is the most likely diagnosis? A Achilles tendon rupture B Iselin's disease C Plantar fasciitis D Sever disease

Correct Answer ( D ) Explanation: Sever disease, also known as calcaneal apophysitis is believed to be a traction apophysitis of the Achilles tendon at the calcaneal apophysis related to overuse. It is most commonly seen in children ages 8-12 years and the pain is often associated with sports that use cleats. Positive physical exam findings include pain with medial-lateral compression of the calcaneus (positive compression test) or pain with direct palpation over the apophysis. Radiographs are not required to make the diagnosis but may show increased sclerosis or fragmentation of the calcaneal apophysis. X-rays can be used to rule out other causes of heel pain if the diagnosis is unclear. Symptomatic management is the mainstay of treatment including the use of heel cups, heel cord stretching exercises, and rest and nonsteroidal anti-inflammatory drugs as needed. Achilles tendon rupture (A) is rare in children and adolescents and is usually a traumatic injury in which the patient complains of a pop and acute onset of pain. Iselin's disease (B) is a traction apophysitis of the tuberosity of the fifth metatarsal at the attachment of the peroneus brevis tendon. Plantar fasciitis (D) is uncommon in children and adolescents and generally presents as pain over the medial calcaneus which can radiate into the arch. The pain is most prominent with the first steps of the day.

A 72-year-old man presents with worsening low back pain for the past one month. The pain is worse with extension of his back and after walking for several minutes. The pain is relieved with sitting. Physical exam shows that extension of the spine produces pain radiating down his bilateral legs. Which of the following is the most likely diagnosis? A Ankylosing spondylitis B Cauda equina syndrome C Herniated disc D Spinal stenosis

Correct Answer ( D ) Explanation: Spinal stenosis is typically associated with neurogenic claudication. Neurogenic claudication is described as pain that is worse after walking for several minutes and relieved with sitting. Additionally, the pain associated with spinal stenosis is typically described as being worse with back extension. This is caused by osteoarthritis in the lumbar spine. The degenerative changes cause narrowing of the spinal canal and compression of neural structures or the spinal artery. Typically, this is found in patients > 50 years of age. Confirmation of the diagnosis can be achieved with MRI or CT. Treatment includes physical therapy, epidural or facet joint corticosteroid injections, and surgical treatments. Surgical options include spinal decompression, nerve decompression, and spinal fusion. Spinal fusion is considered the last treatment option as this involves joining the vertebra to eliminate range of motion and therefore decrease pain. Ankylosing spondylitis (A) is typically starts in patients < 40 years of age. Signs and symptoms include tenderness over the spine and sacroiliac joints and stooped posture. Cauda equina syndrome (B) presents with neurologic deficits (e.g. saddle anesthesia, urinary retention, decreased rectal tone). A herniated disc (C) typically presents with low back pain that radiates down one of the lower extremities and is often associated with numbness and tingling. One Step Further Question: When is it appropriate to refer to orthopedics? Answer: Neuroclaudication symptoms > 12 weeks. Rapid Review Spinal Stenosis Patient will be older Complaining of low back pain and stiffness when walking that is relieved when leaning forward Diagnosis is made by MRI Most commonly caused by narrowing of the lumbar spinal canal with compression of the nerve roots Treatment is physical therapy, surgery

Which nerve root is affected in a patient with loss of the ankle jerk reflex? A C5 B L4 C L5 D S1

Correct Answer ( D ) Explanation: The S1 nerve root is responsible for the ankle jerk reflex (Achilles reflex). Patients with S1 nerve root lesions will experience a diminished ankle jerk reflex, weakness with foot eversion, numbness along the lateral edge of the foot, and radicular pain along the posterior buttocks extending into the posterior thigh and calf. S1 is one of the most common locations for disc herniation. Disc herniation occurs when the collagenous annulus fibrosis tears, allowing the gelatinous nucleus pulposus to protrude. The C5 (A) nerve root is responsible for the biceps reflex and deltoid strength. L4 (B) is responsible for the knee jerk (patellar) reflex, foot inversion strength, sensation in the anterior thigh, and will cause pain in the posterior buttock that wraps around the front of the thigh and into the medial calf and foot. L5 (C) root lesions will present with intact reflexes, great toe dorsiflexion weakness, numbness in the web between the great and second toe, and pain along posterior buttock radiating into the lateral thigh and leg. One Step Further Question: What is cauda equina syndrome? Answer: Damage to the cauda equina causes loss of function of the lumbar plexus (nerve roots) of the spinal canal below the termination (conus medullaris) of the spinal cord

Which of the following is the most reliable indication of an achilles tendon rupture? A Inability to ambulate B Inability to plantar flex the foot C Pain along the posterior ankle D Positive calf squeeze test

Correct Answer ( D ) Explanation: The achilles tendon is formed by the convergence of the gastrocnemius and soleus muscles and attaches on the posterior calcaneus. Achilles tendon ruptures tend to occur when sudden force is placed on the achilles tendon during physical activity that involves pivoting on the foot or sudden acceleration. The majority of ruptures occur in tendons that are already weakened from previous injury or degeneration. Both partial and complete ruptures can occur. Men, typically in their 30s and 40s, are significantly more likely to have an achilles tendon rupture. Patients often present with a history of sudden pain and a "popping" sensation in the posterior ankle during physical activity. On examination, there may be edema and ecchymosis over the achilles tendon and a palpable defect may be appreciated. The calf squeeze test, or Thompson test, is the most reliable indicator of an achilles tendon rupture with a sensitivity of 96%. With the patient prone or kneeling on a chair, the posterior calf is squeezed at its widest point. The absence of plantar flexion indicates a rupture of the tendon. Management includes immobilization with a short leg cast with the ankle in slight plantar flexion, known as the equinus position, and orthopedic consultation. Because of the multiple muscles involved in plantar flexion of the foot that remain intact and functional even with an achilles tendon rupture, the ability to ambulate (A) and plantar flex the foot (B) does not reliably exclude an achilles tendon rupture. Pain along the posterior ankle (C) is typical, however one study showed that up to a third of patients with tendon rupture do not report pain.

A 45-year-old man presents with tenderness on his right elbow that has worsened over the past 24 hours. On physical examination, his right elbow is erythematous, tender and warm to palpation. Range of motion is significantly reduced in the right elbow compared with the left elbow. Which organism is the most common etiology for this disease? A Escherichia coli B Group A streptoccocus C Pseudomonas aeruginosa D Staphylococcus aureus

Correct Answer ( D ) Explanation: The acute onset of monoarticular joint pain, erythema, heat, and immobility should raise suspicion of septic arthritis. Constitutional symptoms such as fever, chills, and rigors are poorly sensitive for septic arthritis. Because the clinical presentation of septic arthritis may overlap with other causes of acute arthritis, arthrocentesis is needed to identify the causative infectious agent. Synovial fluid should be evaluated at the bedside and then sent for white blood cell count with differential, crystal analysis, Gram stain, and culture. Staphylococcus aureus is the organism most commonly found in patients with septic arthritis in the United States and other developed countries. Empiric intravenous antibiotic treatment of septic arthritis should be based on the organism found in the Gram stain of the synovial fluid, or on the suspicion of a pathogen from the patient's clinical presentation. Treatment options include vancomycin for gram-positive cocci, ceftriaxone for gram-negative cocci, and ceftazidime for gram-negative rods. The duration of therapy in patients with nongonococcal septic arthritis is typically three to four weeks. In addition to antibiotic therapy, evacuation of purulent material is necessary. Streptococcus species (B) is the second most common agent responsible for septic arthritis. Gram-negative bacilli represent approximately 14 to 19 percent of septic arthritis cases and are associated with invasive urinary tract infections, intravenous drug use, older age, compromised immune system, and skin infections. The two most common gram-negative organisms detected in adults are Pseudomonas aeruginosa (C) and Escherichia coli (A). One Step Further Question: What is the treatment of choice for disseminated Neisseria gonorrhoeae septic arthritis? Answer: Ceftriaxone

A patient presents with ankle pain after twisting it while walking. Which of the following clinical findings necessitates an ankle X-ray? A Age >50 B Tenderness to the anterior edge of the lateral malleolus C Tenderness to the anterior edge of the medial malleolus D Tenderness to the posterior edge of the lateral malleolus

Correct Answer ( D ) Explanation: The patient presents with an injury suspicious for an ankle sprain and the decision to perform an X-ray can be guided by the Ottawa Ankle Rule. Ankle sprains refer to ligamentous injury to the ankle. In patients presenting with ankle pain after trauma, it is important to determine the presence or absence of a fracture as this may guide management. The Ottawa Ankle Rule was developed to reduce the number of X-rays for ankle injuries. Studies have found it to have a sensitivity near 100%. Application reduces the use of X-ray by 36%. Below are the components of the rule. Therefore, tenderness to the posterior edge of the lateral malleolus necessitates an X-ray. There is no specific age cutoff (A) for the Ottawa Ankle Rules and it has been successfully applied in children over the age of 6. Tenderness anterior to the lateral malleolus (B) is not a criterion for an X-ray and this is commonly seen in patients who sprain the anterior talofibular ligament (ATFL). Tenderness to the anterior edge of the medial malleolus (C) is also commonly seen in ankle sprains. One Step Further Question: What is a Maisonneuve fracture? Answer: A spiral fracture of the proximal 1/3 of the fibula associated with a tear of the distal tibiofibular syndesmsosis and the interosseous membrane

A 40-year-old man who plays weekend baseball as a pitcher presents to the office stating that for the last few months his right shoulder feels like it is "going dead." He states that the pain is progressively worsening and now he is experiencing weakness with overhead activity. He also believes that he is throwing the baseball slower than previously. Which of the following is the most likely diagnosis? A Acromioclavicular joint injury B Adhesive capsulitis C Glenohumeral joint dislocation D Rotator cuff tear

Correct Answer ( D ) Explanation: The rotator cuff is made up of the Supraspinatus (abduction), Infraspinatus (external rotation), Teres minor (external rotation), and Subscapularis (internal rotation) muscles. Often remembered by the mnemonic SITS. Rotator cuff impingement and tearing usually begin in the supraspinatus tendon as it passes under the acromion. Patients are usually >50 years of age and will often have significant pain with abduction above the head and internal rotation (reaching up the back). Often times, patients will complain of difficulty brushing their hair or have pain at night when rolling onto their shoulder. The condition can also occur in young patients, particularly baseball pitchers who will often hear a "pop" caused by a tear in the rotator cuff. On exam, the drop arm test and empty can test are positive. If there is weakness on exam and lack of full improvement with rehabilitative exercises and subacromial corticosteroid injection, you should suspect a tear rather than isolated impingement. Tears are diagnosed by MRI and often require surgical repair. Acromioclavicular injury (A) may be traumatic (fall onto the lateral aspect of the shoulder) or atraumatic (arthritis). A fall on the lateral shoulder can result in a stress or tearing of the acromioclavicular (AC) ligaments resulting in a shoulder separation (not the same as dislocation, which involves the glenohumeral joint). Degenerative arthritis of the AC joint is often seen in patients >50 years of age. Adhesive capsulitis (B), also known as a frozen shoulder, is usually the result from prolonged immobility due to another shoulder injury. It leads to loss of both active and passive motion of the shoulder that usually resolves over a period of 6 months to 2 years. Glenohumeral joint dislocation (C) usually occurs as a result of trauma or from extreme abduction and external rotation (serving a volleyball). On exam, there is often a defect seen at the glenohumeral joint. One Step Further Question: What is the Neer test? Answer: Exam of the shoulder where the examiner performs maximal passive forward flexion with internal rotation while stabilizing the patient's scapula with the other hand. Rapid Review Rotator Cuff Impingement and Tear Supraspinatus (abduction) Infraspinatus (external rotation) Teres minor (external rotation) Subscapularis (internal rotation) + Neer and + Hawkins Pain with brushing hair or teeth Pain at night when rolling onto shoulder Baseball pitchers

A 45-year-old woman presents with nonradicular, flexion-based lower back pain that began atraumatically three weeks ago. Her medical history is only significant for hypertension and nephrolithiasis. She denies numbness, weakness, fever, bowel or bladder changes or a history of cancer. Physical exam is unremarkable. Which of the following is the next best step? A Order a lumbar MRI with contrast B Order a neurosurgical consultation for lumbar spine evaluation C Recommend bed rest for five days D Recommend continued activity within the limits of the pain

Correct Answer ( D ) Explanation: This patient has acute, nonspecific back pain (lumbago). Lower back pain affects 60% to 80% of adults at some point in their lives. Most cases resolve within a few weeks so management is generally conservative. Patients should be instructed to return to work and continue with daily activity based on their pain tolerance. Engaging in activity within the limits of pain aids recovery. Red flags that raise suspicion for a less benign process include night pain and weight loss (suspect neoplasm); fevers, chills, and sweats (spinal infection); acute bony tenderness (fracture); morning stiffness lasting >30 minutes in young adults (seronegative spondyloarthropathy); and any neurologic deficit or bowel or bladder involvement (nerve root compression). Imaging (A) for nonspecific low back pain is low yield and costly and is not recommended. Surgical consultation (B) should be considered if a patient fails conservative treatment, or develops progressive and functionally limiting neurological symptoms such as leg weakness or bowel or bladder incontinence. Prolonged bed rest greater than two days (C) does not hasten healing and may lead to a worse prognosis. One Step Further Question: What other treatment would you consider for this patient with lumbago? Answer: Physical therapy, acetaminophen or NSAIDs, and muscle relaxers

A soccer player presents to the emergency department after being kicked in the shin by his opponent an hour ago. Upon presentation he is diaphoretic and screaming in pain. You assess his lower extremity range of motion and notice that he has extreme pain with passive range of motion of his foot. He complains of numbness and has 1+ distal pulses on the affected limb. What is the best treatment for this patient? A Admit to orthopedics for observation B Increase pain medications C Splint the leg D Surgical fasciotomy

Correct Answer ( D ) Explanation: This patient presents with acute compartment syndrome, and the best treatment for this patient is an immediate surgical fasciotomy. Compartment syndrome occurs when there is an elevated amount of pressure within a confined muscle compartment that can lead to decreased circulation and therefore tissue ischemia. Patients with compartment syndrome present with pain out of proportion to exam, pain with passive range of motion, and sometimes neurologic symptoms such as paresthesia or paralysis. Distal pulses may be diminished or absent. Acute compartment syndrome is a surgical emergency and a fasciotomy must be performed as quickly as possible. A fasciotomy is a procedure performed to open the fascia in order to release the pressure within the affected compartment. This is the only effective treatment for acute compartment syndrome. Admitting to orthopedics for observation (A) will delay time to perform fasciotomy and may lead to limb loss or death. In acute compartment syndrome, it is necessary that the patient be sent directly to the operating room to decompress the affected area. Increasing pain medications (B) will not help to treat the disorder. In patients with acute compartment syndrome, their pain is typically out of proportion to their injury, and an increase in pain medication will not help to decrease their symptoms or fix the source of their pain. Splint the leg (D) is the opposite of what should be done in cases of acute compartment syndrome. Any compressive devices, splints, casts, or other restrictive clothing that would cause any increase in pressure to the extremity should be removed. One Step Further Question: In what time frame should a surgical fasciotomy be performed in order to be most effective? Answer: Six hours from onset of injury.

A patient presents with back pain. Radiographic findings include a herniated nucleus pulposus between L5 and S1. What are you most likely to find on physical examination if the patient has an S1 radiculopathy? A Decreased lateral leg sensation B Urinary incontinence C Weak dorsiflexion of the foot D Weak plantarflexion of the foot

Correct Answer ( D ) Explanation: Weak plantarflexion of the foot is a physical examination finding specific for an L5-S1 herniated nucleus pulposus. A herniated nucleus pulposus, also known as a herniated disc, occurs when there is a weakness in the outer annulus fibrosus and the inner nucleus pulposus herniates outward through the small tears. If not corrected, the nucleus pulposus continues to push outward causing impingement on the spinal nerves resulting in radiculopathy, or disease of a nerve. When a disc herniation occurs between the L5 and S1 vertebra, an S1 radiculopathy occurs resulting in pain radiating down the posterior aspect of the leg and foot along with weakness with plantarflexion due to weakness of the gastrocnemius muscle. The ankle reflex is also typically lost. Diagnosis is formulated through physical examination findings as well as confirmation from imaging. Treatment includes pain medications, physical therapy, and surgical interventions, such as a discectomy to remove the herniated nucleus pulposus from the impinged nerve. Decreased lateral leg sensation (A) is a finding of a nerve impingement at the L4-L5 level. An L5 nerve root impingement is one of the most common sites of radiculopathy of the spine. Patients present with low back pain that radiates down the lateral aspect of their leg and foot and can result in foot drop or decreased strength of foot dorsiflexion. Reflexes are typically spared with an L5 radiculopathy. Weak dorsiflexion of the foot (C) is a result of a herniated nucleus pulposus between L4 and L5. At this level, an L5 radiculopathy causes weakness in foot dorsiflexion, inversion, eversion, and toe extension. Weakness of the foot dorsiflexors results in foot drop because the patient can't lift the foot up. Urinary incontinence (B) is an alarming sign of sacral radiculopathy that is known as cauda equina syndrome. This is a rare, but emergent process in which extreme pressure or swelling compresses the nerve roots intrathecally. Nerves may be compressed at a higher level within the lumbar spine, but the amount of pressure causes symptoms of decreased perineal sensation, as well as urinary and bowel incontinence and sexual dysfunction. This is a life-threatening emergency, and treatment includes emergent decompression of the impinged nerves. One Step Further Question: What is a special test used to elicit radicular nerve pain? Answer: The straight leg raise test

A 17-year-old boy presents to the emergency department after being tackled while playing football. He is unable to bear weight on his left ankle. On physical exam, he is tender to palpation over the anterior tibiofibular ligament. Dorsiflexing while externally rotating the ankle reproduces his pain. Plain radiographs reveals a diastasis between the distal tibia and fibula, but no fractures. Which of the following is the most likely diagnosis? A Achilles tendon sprain B Anterior talofibular ligament sprain C Lateral collateral ligament sprain D Syndesmotic ankle sprain

D Syndesmotic ankle sprain Correct Answer ( D ) Explanation: This boy most likely has a syndesmotic ankle sprain, or high ankle sprain. A syndesmotic sprain involves a ligamentous injury to one or more of the distal tibiofibular syndesmosis. The ankle joint is made up of the tibia, fibula, and the talus. The fibula and tibia are held together by tibiofibular syndesmosis. High ankle sprains are less common than lateral ankle sprains. Risk factor for high ankle sprains are male gender, contact sport participation, and playing on artificial turf. High ankle sprains most commonly occur as the result of external rotation on a planted foot. An antalgic gait or inability to bear weight, ankle edema, and tenderness over the anterior tibiofibular ligament are the most common findings. Physical exam should also include palpation of the proximal fibular, distal tibia and fibula, and talus to evaluate for associated fracture. Special tests including the single-leg hop test, squeeze test, dorsiflexion-external rotation test, and Cotton test, can be performed to further assess the syndesmosis. Plain radiographs are usually indicated in suspected high ankle sprains. Ankle radiographs may demonstrate diastasis between the tibia and fibula depending upon the severity of the sprain. Magnetic resonance imaging is the gold standard imaging modality for visualizing injury to the syndesmosis. Sprains are graded from 1 to 3. Orthopedic referral is indicated for patients with grade 2 or 3 syndesmosis injuries. Grade 1 injuries are managed conservatively with immobilization, crutches, ice, and nonsteroidal anti-inflammatory drugs (NSAIDs) or acetaminophen. Stage 2 and 3 syndesmosis injuries may require operative management due to high risk of chronic pain and disability.

A 62-year-old woman presents complaining of ankle pain following a fall from standing onto the ice last night. She is able to walk across the Emergency Department and back, but states that the pain becomes unbearable toward the end. Her posterior lateral malleolus and her anterior medial malleolus are tender to palpation. Which of the following is the correct indication for ankle radiography in this patient? A Age B Inability to bear weight after several steps C Tenderness over the anterior medial malleolus D Tenderness over the posterior lateral malleolus

D Tenderness over the posterior lateral malleolus

What is the most common extra-articular manifestation of ankylosing spondylitis? A Aortic regurgitation B Inflammatory bowel disease C Psoriasis D Uveitis

D Uveitis Correct Answer ( D ) Explanation: Ankylosing spondylitis is a chronic inflammatory disease that primarily affects the spine and pelvis. With a propensity of affecting men in their 20s and 30s, symptoms include low back pain and stiffness that is insidious in onset, worse in the morning and improves with mild activity. In addition to pain and stiffness in the spine and pelvis, patients often have extra-articular manifestations as well, the most common of which is unilateral uveitis. Occurring in up to 40% of patients with ankylosing spondylitis, uveitis presents with acute onset of unilateral eye pain, photophobia, and blurred vision. Patients with ankylosing spondylitis are also at increased risk of aortic regurgitation (A) likely due to an inflammatory process also affecting the aortic root. Inflammatory bowel disease (B) is seen in about 10% of patients with ankylosing spondylitis. Those patients with ankylosing spondylitis and associated psoriasis (C) tend to have more involvement of peripheral joints and may have a more serious course than those without psoriasis. One Step Further Question: What is the first-line pharmacologic treatment of ankylosing spondylitis? Answer: Nonsteroidal anti-inflammatory medications. Rapid Review Ankylosing Spondylitis Patient will be a male Complaining of low back pain that's worse in the morning and improves with exercise Xray will show multiple vertebral fusions (bamboo spine) Labs will show increased ESR Most commonly caused by presence of HLA-B27 Treatment is NSAIDs, physical therapy Comments: diseases associated with HLA-B27 PAIR: Psoriatic arthritis, Ankylosing spondylitis, Inflammatory bowel disease, Reactive arthritis

A previously healthy 13-year-old girl presents to your office after a positive scoliosis screening at school. X-ray results reveal a Cobb angle of 15 degrees. Which of the following is the most appropriate next step in management? A Bracing B Reassurance that no follow-up is necessary C Surgical referral D Watchful waiting and 6 month follow-up

D Watchful waiting and 6 month follow-up Correct Answer ( D ) Explanation: Scoliosis is a lateral curvature of the spine. Idiopathic scoliosis is the most common type of scoliosis seen by orthopedic surgeons and often presents in adolescence. Adolescent idiopathic scoliosis is defined as a Cobb angle greater than or equal to 10 degrees, age of onset 10 years or greater and no identifiable etiology. The Cobb angle is determined on radiography and is the measurement used to monitor the progression of scoliosis. As patients mature, skeletal curves can progress. Assessment of risk factors for progression affects management, however completely accurate prediction is impossible. Risk factors include age less than 12 years, onset prior to menarche, curves with a Cobb angle greater than or equal to 20 degrees, female sex, and double or thoracic curves. Cobb angle measurements also guide management of patients with scoliosis. Patients with a Cobb angle of 10-19° should be observed and followed clinically every 6-9 months until they reach skeletal maturity. Bracing (A) is recommended for patients with a Cobb angle of 30-39°, although some patients and families will choose bracing with a Cobb angle of 20-29°. Reassurance that no follow up is necessary (B) is correct for patients with a Cobb angle less than 10°, as this does not meet the definition for scoliosis. Surgical referral (D) is indicated for patients with a Cobb angle greater than or equal to 40°. One Step Further Question: Which grading system is used to assess skeletal maturity? Answer: Risser classification system.

A 35-year-old man presents to the ED after injuring himself while playing basketball. He states he felt a pop in his right lower extremity while jumping for a rebound. He can ambulate, but with a limp. On exam, squeezing his right calf elicits no plantar flexion of his foot. What is the appropriate disposition for this injury? A Admission to hospital for serial compartment pressure measurements B Elastic wrap, crutches, weight-bearing as tolerated, and orthopedic follow-up within seven days C Splint in dorsi-flexion, keep nonweight-bearing, orthopedic follow-up D Splint in plantar-flexion, keep nonweight-bearing, orthopedic follow-up

D. Splint in plantar-flexion, keep nonweight-bearing, orthopedic followup Correct Answer ( D ) Explanation: The patient has an Achilles tendon rupture suggested by the inability of his foot to plantar flex with squeezing of the calf (Thompson test). This injury is most common in middle-aged recreational athletes. Patients often note a "popping" sensation followed by acute weakness and the inability to continue activity. A defect may be palpable on examination of the distal Achilles tendon. Management in the ED includes immobilization of the extremity in a short-leg splint, keeping the foot in plantar-flexion. The patient should be nonweight-bearing until seen by an orthopedic surgeon. Definitive repair is surgical. This injury is not at risk to develop compartment syndrome (A). An elastic wrap (B) alone is insufficient. In an effort to maintain approximation of the proximal and distal aspects of the ruptured tendon, the foot should be splinted in plantar, not dorsi-flexion (C).

Which of the following is used to diagnose and monitor scoliosis? ACobb angleCorrect Answer BHoffman reflex CModified Schober test DPelvic tilt Correct Answer ( A )

Explanation: Scoliosis is lateral curvature of the thoracic or lumbar spine > 10 degrees. It is usually idiopathic, but can be secondary to neuromuscular disease (cerebral palsy, spinal muscular atrophy, myelomeningocele), vertebral disease (tumor, infection), neurofibromatosis, or Marfan syndrome. Symptoms usually develop in early adolescence. Females are more likely to have progressive disease requiring treatment. Adult onset scoliosis is commonly caused by spinal degenerative disk disease, facet spondylosis and spondylolisthesis. On exam, use a forward bending test to assess for vertebral and rib rotation. The diagnosis is made radiographically with weight-bearing PA and lateral full-length spinal views to measure the Cobb angle. The Cobb angle is used to quantify the magnitude of spinal deformities. To measure the Cobb angle, first decide which vertebrae are the end-vertebrae of the curve deformity (vertebrae at the upper and lower limits of the curve) and then Cobb angle formed by the intersection of two lines: one parallel to the endplate of the superior end vertebra and the other parallel to the endplate of the inferior end vertebra. The angle is plotted and scoliosis is defined as a lateral spinal curvature with a Cobb angle of 10° or more. The Hoffman reflex (B) involves assessing for thumb or second digit flexion when the nail of the third digit is flicked. A positive response suggests upper motor neuron disease such as cervical spinal cord injury. The modified Schober test (C) is used to quantify lumbar spine flexion and extension in the sagittal plane. It is not used to monitor worsening coronal spine curve as in progressive scoliosis. Pelvic tilt (D) refers to the height difference between the iliac crests. It is primarily used to evaluate leg-length discrepancy. One Step Further Question: Name some neuromuscular causes of childhood secondary scoliosis? Answer: Spina bifida, cerebral palsy, tethered cord syndrome, muscular dystrophy and spinal muscular atrophy. Rapid Review Scoliosis Lateral curvature of the spine >10 degrees F > M Causes: Idiopathic > neuromuscular disease Forward bending test: assesses vertebral and rib rotation Cobb angle: quantifies magnitude of deformity

A 24-year-old man presents with right shoulder pain after a fall onto his outstretched hand. A radiographic demonstrates an anterior inferior articular glenoid fracture. What is the name of this radiographic finding? A Bankart lesion B Hill-Sachs lesion C Labral lesion D Luxatio erecta

Correct Answer ( A ) Explanation: A Bankart lesion is secondary to an impaction injury of the posterosuperior aspect of the humeral head into the anteroinferior glenoid rim causing a fracture. It is often associated with an anterior glenohumeral dislocation from a force (often falling) with the arm in abduction and external rotation. It is best seen on CT; however, it can be identified by plain radiographics. A Hill-Sachs lesion (B) is a fracture of the posterosuperior humeral head. A labral lesion (C) refers to an injury of the glenoid labrum. Luxatio erecta (D) is characterized by inferior and sub glenohumeral dislocation. One Step Further Question: What is a reverse Bankart lesion? Answer: A fracture of the posterior glenoid rim from a posterior glenohumeral dislocation.

A 44-year-old previously healthy man presents to the Emergency Department with low back pain that started two days ago. He recently moved to a new apartment and has been lifting boxes and furniture. He denies any bowel or bladder incontinence. On examination, he is afebrile. He has tenderness with palpation of the right lumbar paraspinal muscle which radiates to the buttock. Reflexes and strength are intact. Pain is exacerbated with movement. What is the next best step in management? A Five day course of oral corticosteroids B Magnetic resonance imaging of the lumbosacral spine C Plain radiographs of the lumbosacral spine D Prescribe ibuprofen and discharge home

D Prescribe ibuprofen and discharge home Correct Answer ( D ) Explanation: Patients with uncomplicated musculoskeletal low back pain present with pain of the lumbar paraspinal muscles which can radiate to the buttock or posterior thigh. Pain is worse with movement of the trunk, but improves with rest. Strength, sensation, and reflexes will be intact and there will be no history of bowel or bladder incontinence. Presence of any of these neurologic findings on examination would be considered a red flag and should prompt further workup with imaging. Management of uncomplicated low back pain includes oral nonsteroidal anti-inflammatory medications (e.g. ibuprofen) and discharge home with primary care follow-up. Oral narcotic medications can be used in the short-term for breakthrough pain, but have increased side effects and are not considered first line treatment. Strict bed rest is no longer recommended as patients who remain active have an earlier resolution of symptoms. A five day course of oral corticosteroids (A) has not shown to be significantly beneficial in treatment of uncomplicated back pain. Given this patient has no concerning findings on history or physical examination, MRI of the lumbosacral spine (B) and plain radiographs of the lumbosacral spine (C) are not indicated. One Step Further Question: What are the two most common primary sources of spinal metastatic disease? Answer: Lung and breast cancer.

A 45-year-old woman presents with low back pain. She states that she has had back pain for years but it is worse today. She reports pain in the low back that radiates to her left foot. Examination reveals no midline tenderness to palpation and a positive straight leg raise. She has no weakness and a normal sensory exam. What diagnostic testing is indicated in the Emergency Department? CT scan of the lumbar spine MRI of the lumbar spine No immediate imaging X-ray of the lumbar spine

No immediate imaging Correct Answer ( C ) Explanation: This patient presents with signs of symptoms of sciatica, which may be due to a herniated disk and does not require emergent imaging. Sciatica describes a lumbar radiculopathy that is common seen in individuals. Patients typically complain of sharp, burning or shooting pain beginning in the back and radiating down the leg past the knee. There may be associated numbness or weakness or both on the affected side. The pain is often exacerbated with bending, straining or sitting and relieved with lying supine. Physical examination often reveals tenderness in the sciatic notch and a positive straight leg raise. A straight leg raise is performed with the patient lying supine. The symptomatic leg is then passively raised with the knee fully extended. Eliciting back pain radiating down the leg into the knee at 30 - 70 degrees of leg elevation is suggestive of an L5-S1 radiculopathy. The straight leg raise has a good sensitivity (91%) but poor specificity (26%). Sciatica symptoms are typically caused by lumbar disk herniation (sensitivity of 95%). Weakness of ankle dorsiflexion, toe extension, ankle plantar flexion and knee extension are common findings in disk herniation. Immediate imaging is not indicated in patients with symptoms consistent with disk herniation. The majority of patients with disk herniation and sciatica will improve with conservative management alone. CT scan (A) is an excellent modality for evaluating the presence of fractures of the vertebral column but in the absence of trauma, the yield to this test is low. MRI (B) is the definitive modality in the evaluation of possible cauda equina, spinal infection or malignancy but the patient's presentation is not consistent with these diagnoses. However, the patient in the above clinical scenario does not have any sensory or motor complaints. Plain radiographs (D) have little role in the evaluation of low back pain as they contribute little to the management of these patients.


Kaugnay na mga set ng pag-aaral

Chapter 4: Newton's Second Law of Motion

View Set

#1 Chapter 37: Assessment and Management of Patients With Allergic Disorders

View Set

Fluid & Electrolytes Prep U Questions

View Set

Medical Law and Ethics Quiz 5-8 *

View Set

The Computer and Technology Revolutions.

View Set

Computer Applications Cumulative Exam (90%)

View Set

PSYC-305 Final Exam Review Part 3

View Set

Chapter 2.1: Drawing InQuisitive

View Set